Сохранен 510
https://2ch.hk/sci/res/236711.html
24 декабря Архивач восстановлен после серьёзной аварии. К сожалению, значительная часть сохранённых изображений и видео была потеряна. Подробности случившегося. Мы призываем всех неравнодушных помочь нам с восстановлением утраченного контента!

Тред тупых вопросов № 18

 Аноним 03/02/15 Втр 13:50:00 #1 №236711 
14229606009170.jpg
А мы продолжаем познавать Вселенную вместе с лучшими анонимными учеными уровня /sci, а так же задорно и весело унижать лалок, не знающих, что ток возникает в проводнике со скоростью распространения э.м. взаимодействия в данной среде.

Тред тупых вопросов № 18. Вы знаете, что надо делать.
Прошлый тред потихоньку идет ко дну здесь- >>230513
Аноним 03/02/15 Втр 13:57:34 #2 №236714 
Раньше, давно, давно, все взаимодействия были объединены в одно. Довольно быстро отвалилась гравитация, потом сильное и со временем электромагнитное и слабое. Так вот, а может ли в дальнейшем появится еще какое- нибудь взаимодействие?
sageАноним 03/02/15 Втр 17:17:23 #3 №236767 
>>236714
>Раньше, давно, давно, все взаимодействия были объединены в одно.
Откуда ты знаешь? Ты был тогда?
>Довольно быстро отвалилась гравитация, потом сильное и со временем электромагнитное и слабое.
Охуительные истории.
>а может ли в дальнейшем появится еще какое- нибудь взаимодействие?
Только если между мной и твоей мамкой.
Аноним 03/02/15 Втр 17:50:00 #4 №236772 
>>236767
Пшел отсюда.
Аноним 03/02/15 Втр 17:56:37 #5 №236774 
>>236714
Хуй знает. Теоретически, понижение калибровочной симметрии стандартной модели можно было бы продолжить (средуцировать нахуй SU(3) и SU(2) к меньшим группам). В реале -- вряд ли. Уж куда ниже-то по энергиям.
Аноним 03/02/15 Втр 18:01:28 #6 №236779 
Посоны поясните за спин, что это за зверь такой?
Аноним 03/02/15 Втр 18:38:04 #7 №236794 
>>236779
Упрощенная трактовка: (для большинства частиц, за исключ. безмассовых) Собственный угловой момент частицы в собственной системе отсчета. Квантуется. Может, в отличие от обычного орбитального момента, быть полуцелым (в единицах Планка).
Аноним 03/02/15 Втр 19:25:07 #8 №236813 
>>236714
Вот кстати то значит были объединены в одно? Был единственный(или не единственный) бозон-переносчик взаимодействия? Или какая-то другая хуйня? Объясните.
Аноним 03/02/15 Втр 19:47:37 #9 №236816 
>>236813
>Был единственный(или не единственный) бозон-переносчик взаимодействия?

Типа того. Точнее, была единая константа связи и единая группа симметрий этого взаимодействия, и набор отвечающих ей бозонов-переносчиков. Короче, без матана калибровочной теории поля тут сложно адекватно объяснить.
Аноним 03/02/15 Втр 22:20:13 #10 №236829 
>>236711
кто все эти люди на оппике?

кроме галилея, кюри, ньютона, хокинга, эйнштейна, тайсона, докинза и дарвина
Аноним 03/02/15 Втр 22:22:18 #11 №236830 
Канон пояснил по хардкору. Есть две ракеты. Они вылетают из скорости
точки. Скорость каждой 0,6 скорости света. Почему скорость удоления ракет друг от друга будет меньше скорости света.
Аноним 03/02/15 Втр 22:24:42 #12 №236831 
>>236830
Анон поясни по хардкору
Вылетают из одной точки
Пофиксил. Сорри подвыпит.
Аноним 04/02/15 Срд 04:40:03 #13 №236868 
>>236830
> почему
потому что именно друг от друга
Аноним 04/02/15 Срд 06:23:52 #14 №236875 
>>236830
Потому что релятивистский закон сложения скоростей отличается от обычного.
v_отн=(v1+v2)/(1+v1v2/c^2)
Выводится из преобразований Лоренца.
Аноним 04/02/15 Срд 09:02:25 #15 №236879 
>>236875
Пруфы экспериментов то есть?
Аноним 04/02/15 Срд 09:04:01 #16 №236880 
>>236879
Нет конечно
Аноним 04/02/15 Срд 09:51:58 #17 №236883 
14230327182430.png
>>236879
пруфушок закукарекал
Аноним 04/02/15 Срд 12:45:01 #18 №236904 
>>236879
В коллайдере.
Аноним 04/02/15 Срд 15:01:48 #19 №236926 
>>236830
Так скорость удаления друг от друга, приближение друг к другу, скорость тени, скорость солнечного зайчика может превышать скорость света. Ограничение скорости света только для собственной скорости, скорости самого объекта
Аноним 04/02/15 Срд 16:08:15 #20 №236932 
>>236926
>Так скорость удаления друг от друга

Смотря что под ней понимал анон.
Если как стандартную относительную скорость (т.е. одной ракеты относительно другой в её СО) - тогда не выше с. Ф-ла сложения скоростей >>236875 именно для этого случая пишется.
А если как скорость взаимного разбегания ракет относительно неподвижной СО (т.е. dl/dt, где l - расстояние между ними, измеренное в покоящейся СО, t - время по часам той же СО) - тогда, конечно, может превышать с, как с зайчиками. Только о такой скорости обычно не говорят, потому что это не есть скорость какого-либо конкретного объекта относительно какой-либо СО, а чисто формальная величина.
Аноним 04/02/15 Срд 16:14:59 #21 №236933 
Поясните по хардкору о замедлении времени. Почему время ускоряется для близнеца в ракете , а не наоборот? Всё от массы зависит? И как это замедление Выглядит в общей системе (Галактика или кластер)?
Аноним 04/02/15 Срд 16:29:40 #22 №236934 
>>236933
По хардкору - у двух близнецов разные длины мировых линий. Близнец в ракете, чтобы вернуться на Землю и зафиксировать расхождение часов, должен испытать ускорение. Это нарушает инерциальность его СО по отн. к близнецу на Земле => теряется симметрия между близнецами. Это всё, конечно, слова. Гугли точный расчет в любом учебнике по СТО, где разбирается этот эффект.
Аноним 04/02/15 Срд 16:47:57 #23 №236937 
>не знающих, что ток возникает в проводнике со скоростью распространения э.м. взаимодействия в данной среде.

Зря ты об этом напомнил, няша.
Возникает вдоль проводника -- да, со скоростью с.
Течет (в смысле упорядоченного движения носителей) -- со скоростью ~мм/c.
(Быдло)электрики общепринято понимают под скоростью тока скорость его возникновения.
Нормальные физики пожимают плечами и не спорят о дефинициях (держа в уме \vec j=\rho \vec v)
Аноним 04/02/15 Срд 17:35:28 #24 №236944 
>>236937
А вот и порватка из прошлого треда.
Аноним 04/02/15 Срд 18:36:02 #25 №236952 
>>236937
Упорядоченное движение носителей, т.е. ток, распространяется по проводнику со скоростью света, сами носители- со скоростью ~мм/c.
Угомонись уже, физик ты наш, серьезно. Или тебе просто нравится унижаться?
Аноним 04/02/15 Срд 19:12:32 #26 №236960 
>>236952
>не спорят о дефинициях
ок-ок
Аноним 04/02/15 Срд 19:19:53 #27 №236962 
>>236960
>это мелкобуквенное быдло
Ясно.
Аноним 04/02/15 Срд 19:25:33 #28 №236963 
>>236962
>быдло
>порватка
>не может угомониться и засирает тред детскими никому в хуй не впершимися обзывалками
>придирается к регистру
>считает, что очень грамотно ведет дискуссию

Пшел нахуй с треда, флеймер. Выглядишь, как кретин, чесслово, независимо от сути спора.
Аноним 04/02/15 Срд 19:30:42 #29 №236964 
>>236963
Маня, прекрати унижаться. Я конечно понимаю, что у тебя БОЛЬ от того, что ты так глупо обосрался с током, но все- таки.
Аноним 04/02/15 Срд 19:33:00 #30 №236966 
>>236964
>Маня
>обосрался
>БОЛЬ
-> >>236963

Аноним 04/02/15 Срд 19:35:13 #31 №236967 
>>236966
>ВРЕТИИИ!!11 Я НИ АБАСРАЛСЯ ВЫ ВСЕ ДУРАКИ ФИЗИКИ НИ ЗНИТИ Я НАСТАЯЩИЙ ФИЗЕК!111
As expected.
Аноним 04/02/15 Срд 19:38:44 #32 №236969 
>>236967
Бля, да угомонись уже. Пластинку заело?
Аноним 04/02/15 Срд 19:42:31 #33 №236973 
>>236969
>ВРЕТИИ!!1 ЭТА ШАКАЛАД Я НАСТАЯЩИЙ ФИЗЕК И Я НИ АБАСРАЛСЯ.
Успокойся уже, физик.
Аноним 04/02/15 Срд 20:00:14 #34 №236982 
/sci/, что из себя представляет электрический заряд? Википедия и учебники физики говорят что это физическая величина определяющая способность тел ... бла бла бла. То есть ясно, что заряд - мера НЁХ, но что такое эта НЁХ?
Аноним 04/02/15 Срд 20:27:42 #35 №236991 
>>236982
>заряд - мера НЁХ
This. Если же тебя смущает именно "НЁХ", то можешь пройти обратно в пещеру, там же все такое простое и понятное, никаких НЁХ. По большому счету разница между НЁХ и не-НЁХ только психологическая- одно привычно, другое- нет. В науке же критерий того, что что- то не является НЁХ- это наличие математического описания закономерностей поведения и связи с уже известными явлениями. Все. Остальное сугубо личные тараканы каждого.
Аноним 04/02/15 Срд 23:33:53 #36 №237047 
>>236816
Ясно. А то мне казалось что великое объединение это когда все 4 взаимодействия есть, но из-за высокой энергии, хуй поймешь какое именно влияет на объект.
Аноним 04/02/15 Срд 23:39:04 #37 №237048 
Я правильно понимаю что поле это когда вещество излучает бозоны это поля по всей сфере, которые улетают в ебеня, если на их пути ничего нету, отдавать энергию и поэтому считается что поле это бездискретная форма материи?
Аноним 06/02/15 Птн 10:16:14 #38 №237146 
>>237048
Ваша мысль не формируется.
Аноним 06/02/15 Птн 11:25:38 #39 №237153 
>>237048
Да, ты понял правильно. Такие частицы зовутся виртуальными. Пока они не взаимодействуют их "как бы" и нет.
Аноним 06/02/15 Птн 11:28:11 #40 №237154 
>>237048
Правда мне пришлось расшифровывать. Ты был пьян?
Аноним 06/02/15 Птн 16:02:36 #41 №237220 
>>237154
Типа того, не спал сутки из-за сессии.
Аноним 06/02/15 Птн 16:09:30 #42 №237222 
Думаю, какой третий язык учить для международной научной арены. Английский знаю, немецкий знаю, думаю, что выучить из азиатских: корейский/китайский/японский.

Не поможете советом?
Аноним 06/02/15 Птн 16:16:23 #43 №237226 
>>237222
Китайский, очевидно же.
Аноним 06/02/15 Птн 16:21:04 #44 №237229 
>>237222
Волапюк
Аноним 06/02/15 Птн 16:30:48 #45 №237235 
>>237229
Забытое и ненужное говно без задач.
Аноним 06/02/15 Птн 16:36:07 #46 №237237 
>>237235
Зато название угарное.
Аноним 06/02/15 Птн 16:36:50 #47 №237238 
>>237226
ОК, буду учить китайский. Все так и советуют.
Аноним 07/02/15 Суб 10:46:50 #48 №237344 
Если разогнать космолет из 2-х соединенных частей до световой скорости, а потом оттолкнуть назад половину что будет? (при обычном движении оставшаяся часть ускорится)
Аноним 07/02/15 Суб 15:07:38 #49 №237368 
Антуаны, почему, когда разбиваешь яйцо, белок как бы разделяется, сначала вытекает жидкая часть, которая, как сперма, а потом густая, соплеобразная? Ведь, в целом яйце нет такого разделения.
Аноним 07/02/15 Суб 15:11:00 #50 №237371 
>>237344
Начнём с того, что самолёт в принципе невозможно разогнать до световой скорости, если только он не из калибровочных бозонов
Аноним 07/02/15 Суб 15:21:10 #51 №237374 
Забыл вопрос.
Аноним 07/02/15 Суб 15:25:08 #52 №237376 
Про космолифт. Для него нужны нанотрубки - каждая должна быть длиной 36000км или можно из обрывков сделать? Он должен утолщаться - трубки тоже надо как-то утолщать или просто новые слои крепить постепенно?
Аноним 07/02/15 Суб 15:30:08 #53 №237378 
>>237376
На все воля божья.
Аноним 07/02/15 Суб 18:32:59 #54 №237399 
3-deoxy-D-manno-octulosonic acid можно перевести как 3-дезокси-D-манно-октолосониковая кислота?
Аноним 07/02/15 Суб 18:52:20 #55 №237403 
>>237371
Так и знал что кто-нибудь это напишет, ну вообрази до 99,999%, какая разница. И пруфануть за невозможность хорошо бы.
Аноним 07/02/15 Суб 19:19:28 #56 №237408 
>>237344
> потом оттолкнуть назад половину что будет?

Суммарный импульс (релятивистский) сохранится => оставшаяся часть слегка ускорится. Но все равно скорость обеих частей не превысит c. потому что релятивистский закон сложения скоростей не наебешь.

>И пруфануть за невозможность хорошо бы.
Пруф тока один может быть - формализм СТО, по которому ровно со скоростью с движутся тока безмассовые частицы. Для разгона массивной частицы до c требуется бесконечная энергия.
Аноним 07/02/15 Суб 20:34:22 #57 №237434 
14233304623520.jpg
Что за Великий аттрактор?
Аноним 07/02/15 Суб 20:48:56 #58 №237438 
14233313361250.jpg
>>237434
гипотетический оче тяжелый объект, по направлению к которому двигаются многие скопления галактик. Двигаются ли они туда с ускорением или просто так совпало - науке это не известно, наука пока не курсе дела. Поэтому Великий Аттрактор - всего лишь гипотенуза.

пик - почти совершенно не релейтед
Аноним 07/02/15 Суб 21:35:11 #59 №237446 
>>237368
Бумп вопросу
Аноним 07/02/15 Суб 21:39:53 #60 №237447 
https://www.youtube.com/watch?v=v2J4lJAvpLM

/sci/ поясни по хардкору, я нихуя не понял
Аноним 07/02/15 Суб 22:36:50 #61 №237462 
>>237408
>СТО
Ну формулы это хорошо, но как это выглядит на практике интересно же.
>Суммарный импульс (релятивистский) сохранится => оставшаяся часть слегка ускорится. Но все равно скорость обеих частей не превысит c. потому что релятивистский закон сложения скоростей не наебешь.
Допустим снимает видеокамера как отстреливается задняя часть, что там будет? Или лучше даже на носу поставить еба пушку и выстрелить, куда снаряд денется? Ну вроде как масса увеличивается, это повлияет? Но как тогда вообще при больших скоростях могут работать механизмы? С другой стороны на любую еба массу найдется еба энергия чтобы ее толкнуть. Может снаряд сохранит скорость, а пушка замедлится?
Аноним 07/02/15 Суб 22:37:31 #62 №237463 
Саентач, я пришел к выводу что случайности в физическом смысле не существует. Возьмем пример с монеткой, если поставить монетку ребром на плоскую поверхность. Если на нее подуть с достаточной силой, причем она всегда будет падать в направлении потока воздуха.

Теперь подбросим монетку, мы точно знаем, что она упадет вниз, однако какой стороной вверх - неизвестно. Когда мы не знаем точного результата какого-либо события, мы говорим что это происходит случайным образом, однако как и в примере с монеткой на столе, на нее действуют вполне определенные силы, по одним и тем же законам, что говорит нам, что если бы мы знали, какие явления действуют на монетку во время броска, и знали бы по каким законам их можно описать, мы смогли бы точно знать, каким образом она упадет. Единственная проблема в том, что колличество вещей, которые воздействуют на монетку стремится к бесконечности. Начиная с притяжения Земли, заканчивая воздействием какого-либо излучения звезды из другой части нашей галактики. Однако большинство этих факторов воздействуют на монету с той малой силой, что практически не влияют на ее траекторию и другие параметры.

Таким образом мы можем сказать, что случайность - это упрощение, вызванное невозможностью формулировки всех законов, которые необходимо применить, чтобы полностью описать все взаимодействия.

Мне без разницы, если это всем уже известно, я просто хочу узнать, прав ли я или где проебался.
Аноним 07/02/15 Суб 22:54:11 #63 №237467 
>>237463
Проёб только в квантовой механике и в теореме о неполноте формальных систем. См. тред про "случайна ли случайность".
Аноним 07/02/15 Суб 23:27:18 #64 №237468 
>>237462
>как это выглядит на практике интересно же.

На практике это выглядит так.
1) Достичь околосветовой скорости помешает всякое межзвездное говно вроде пыли, частиц и пр., столкновение с которыми будет тем фатальней, чем выше скорость.

2) Допустим, все-таки ты достигл скорости в 99,9999с в некой идеальной, свободной от межзвездной среды, вселенной. Равномерно летящий космолет, независимо от своей скорости, будет представлять из себя обычную инерциальную систему отсчета (ИСО). Вся физика в ней согласно принципу равноправия всех ИСО будет абсолютно неотличима от других ИСО: все механизмы на борту и за бортом будут работать как обычно. Не выглядывая за борт, ты никакими механическими опытами не сможешь даже установить или почувствовать, движешься ты или покоишься. Отстреленная камера или снаряд полетят относительно тебя с обычной их скоростью, какую они получили бы, будучи отстреленными от покоящегося корабля.

"Увеличение" массы, замедление времени и пр. изнутри космолета почувствовать нельзя.
Аноним 07/02/15 Суб 23:40:08 #65 №237471 
>>237463
>случайности в физическом смысле не существует

Кагбе есть точка зрения, что кванмех дает пример процессов, которые якобы истинно случайны. Но тут опять же - что понимать под случайностью. Во всяком случае, квантмех согласуется с понятием случайности в операционалистском смысле: никакая сумма знания о состоянии вселенной в какой-то момент не дает операциональной возможности предсказать ход событий.

Другой вопрос, если мы ведем речь о случайности в онтологическом смысле, т.е. отходим от физической эмпирики в область философских категорий причинности, и называем случайностью ситуацию, когда существуют истинно беспричинные события безотносительно к возможностям субъекта-наблюдателя. Т.е каузальный индетерминизм. Вот тут уже квантмех, строго говоря, ничего не решает,это уже вопрос интерпретации и философской установки. Радикальный сторонник копенгагенской интерпретации склонился бы к такому индетерминизму, тогда как сторонник бомовской формулировки счел бы возможным сохранить детерминизм за счет скрытых параметров, но лишь ценою введения нелокальности.

Спорный вопрос, возможен ли каузальный детерминизм (т.е. существование онтологически беспричинных, ничем не обусловленных событий) логически. Во всяком случае, возникает вопрос -- как на уровне логики представить ситуацию, что некоторое событие ничем не обусловлено, а тем не менее в силу чего-то (чего??) происходит так, а не иначе.
Аноним 07/02/15 Суб 23:41:21 #66 №237472 
>>237471
>Спорный вопрос, возможен ли каузальный индетерминизм

быстрофикс
Аноним 07/02/15 Суб 23:53:02 #67 №237473 
>>237468
скорости в 99,9999 процентов с, ессно
Аноним 08/02/15 Вск 00:47:50 #68 №237476 
14233456701660.jpg
Анон, я тут думал о детерменизме, и у меня возник вопрос. Представь, стоит где-нибудь человек, поднявший правую руку. Он может стоять так пять секунд, минуту, час, быть может дольше. Никто и ничто не мешают ему простоять так любой из этих отрезков. И вдруг, в один из моментов времени, он решает опустить руку. Он может сделать это в любой момен, никто и ничто (условно) его не заставляет.
Но, как я понимаю, в детерминистической трактовке мира у всего должна быть причина. В таком случае, что среди атомов и прочей материи в его мозгу и в его теле предсказывало, когда именно он опустит руку? Я думаю, ничто. Другими словами, он совершенно внезапно решил опустить руку. Приказ опустить руку отдало сознание. В таком случае, если материя не содержит в себе причины для принятия этого решения - опускания руки, разве сознание не будет тогда чем-то нематериальным? Что на эту тему говорят нейрофизиологи, физики и прочие учёные? Где находится сознание?
Аноним 08/02/15 Вск 01:47:31 #69 №237483 
>>237476
>И вдруг, в один из моментов времени, он решает опустить руку. Он может сделать это в любой момен, никто и ничто (условно) его не заставляет.
> он совершенно внезапно решил опустить руку. Приказ опустить руку отдало сознание.

Увы, как раз в такой постановке - свободное, ничем не подгоняемое решение - сознание фэйлит по-черному. Эксперименты Либета.

http://elvensou1.ru/svoboda-voli-illyuziya
>Был довольно долгий спор относительно того, что субъективное ощущение «волевого» решения определяется активностью мозга моментами ранее. Мы обнаружили, что результаты решения могут быть закодированы в мозговой активности префронтальной и теменной коры за 10 сек до его появления в сознании. Эта задержка, по-видимому, отражает работу сети областей высокоуровневого контроля, которые начинают готовить предстоящий решение задолго до его осознания.

Аноним 08/02/15 Вск 06:29:55 #70 №237499 
Я только что случайно конденсат Бозе-Эйнштейна, что делать?
Аноним 08/02/15 Вск 10:58:50 #71 №237505 
>>237468
>Достичь околосветовой скорости помешает всякое межзвездное говно вроде пыли, частиц и пр., столкновение с которыми будет тем фатальней, чем выше скорость
Можно сделать какую-нить защиту или перенести эксперимент в межгалактическое пространство, есть почти пустые места. Опять же была бы энергия, способ найдется.
>абсолютно неотличима от других ИСО
Ок.
>Отстреленная камера или снаряд полетят относительно тебя с обычной их скоростью, какую они получили бы, будучи отстреленными от покоящегося корабля.
И с какой разницей они прилетят к месту назначения (финишная ленточка без торможения скажем)? По ото ни с какой, но они же будут далеко друг от друга физически, с точки зрения космонавтов. Не понимат.
Аноним 08/02/15 Вск 12:01:10 #72 №237506 
>>237505
В ранних звездолетах типа "Тахмасиб" перед звездолетом на расстоянии километра летел специальный щит вроде блина из сверхтвердой материи. При встрече с таким щитком космический газ и пыль, микрометеориты просто аннигилровались и превращались в энергию, ибо скорость Тахмасиба приближалась к околосветовой.
В новейших звездолетах применяются энерегетические экраны из пространственных полей, и проблема соударений теряет смысл - поля искривляют пространство и встреча с любым матриальным объектом становится невозможной - любое тело из трехмерного пространства "огибает" звездолет. Или упрощенно, размер звездолета в трехмерном пространстве стермится к нулю пропорциоанально скорости движения в варпе.
Таким образом, современные звездолеты являются надсветовыми кораблями. Есть варп-звездолеты с линейной скоростью, есть нуль-звездолеты с скачкообразным нуль-переходом, а есть гиперпереходники с квантовым скачком.
Аноним 08/02/15 Вск 13:07:02 #73 №237515 
>>237505
Отстреленный объект прилетит с задержкой, обусловленной приобретенной разницей скоростей, как и в обычной механике. Отличие будет только в значениях этих скоростей - каждая будет рассчитана по формулам СТО и не будет превышать с. Разницы бы "не было", если бы космолет летел точно со скоростью света - но это противоречивая постановка, с таким объектом нельзя связать систему отсчета, в ней все линейки и часы делятся на нуль.
Аноним 08/02/15 Вск 13:21:09 #74 №237517 
>>237515
То есть импульс не сохраняется как обычно? Скажем если части равны массой задняя потеряет 1000км/с, а передняя наберет скажем 100км/с еще немного приблизившись к световой? Не понятно тогда как будет работать обычная механика внутри корабля, она же вроде не должна меняться.

>Разницы бы "не было", если бы космолет летел точно со скоростью света - но это противоречивая постановка, с таким объектом нельзя связать систему отсчета, в ней все линейки и часы делятся на нуль.
А нет вариантов разогнать до с хотя бы протон в ускорителе? (хотя бы теоретическая возможность)
Аноним 08/02/15 Вск 13:38:00 #75 №237522 
>>237517
> То есть импульс не сохраняется как обычно?
Закон сохранения импульса в СТО выполняется.
> А нет вариантов разогнать до с хотя бы протон в ускорителе?
Потребуется бесконечная энергия.
Аноним 08/02/15 Вск 13:45:55 #76 №237523 
14233923553090.jpg
>>236711
Здравсвуй тред, я немного мудак.
годное начало
Очень нужна помощь.
дана задача, но я НЕ БУДУ просить решить её за меня. я же не уебан какой, сам не люблю такое делать.
пришёл я не с пустыми руками. просто задачи тут скользкие + я как омега сомневаюсь в себе, мало ли, может решил и без косяков:

Аноним 08/02/15 Вск 14:12:50 #77 №237530 
Анон. Согласно ТО с точки зрения стороннего наблюдателя, внутри корабля движущегося с близкой к свету скоростью время течёт медленно и исходя из этого пассажир кораблика год полёта ощутит как пара недель. Но ведь согласно ТО пространство снаружи кораблика для этого пассажира движется с той же близкой к свету скоростью. Для пассажира время снаружи будет течь медленно. Тогда полёт для этого пассажира покажется ещё длиннее, по идее?
Аноним 08/02/15 Вск 14:16:36 #78 №237533 
>>237530
>Но ведь согласно ТО пространство снаружи кораблика для этого пассажира движется
>согласно ТО пространство движется
>движется
>пространство
>согласно ТО
Что?
Аноним 08/02/15 Вск 14:18:22 #79 №237535 
>>237522
>Закон сохранения импульса в СТО выполняется.
Но как это будет выглядеть то??
>Потребуется бесконечная энергия.
Жалко. Но все же в обычной механике импульсы просто складываются, а по сто? Допустим корабль разгоняется пуляя кирпичи ацкой гауус пушкой запитанной от еба термоядерного реактора. В классике же он будет вечно ускоряться пока кирпичи не кончатся, а что говорит сто на этот счет, как это будет выглядеть? (техническую сторону можно реализовать кстати сделав несколькоступенчатый звездолет)
Аноним 08/02/15 Вск 15:09:34 #80 №237542 
>>237535
>Закон сохранения импульса в СТО выполняется.
>Но как это будет выглядеть то??

Да точно так же: P_общ=P1+P2 (векторно). Только P - не классический, а релятивистский импульс (содержит корень из 1-v^2/c^2), поэтому все скорости соответственно отхерачиваются к масштабу не выше с.
Аноним 08/02/15 Вск 15:14:32 #81 №237545 
>>237542
>В классике же он будет вечно ускоряться пока кирпичи не кончатся, а что говорит сто на этот счет, как это будет выглядеть?

Есть такая тема в СТО - движение равноускоренного объекта. Для внешнего наблюдателя скорость объекта будет асимптотически стремиться к с.
http://ru.wikipedia.org/wiki/%D0%A0%D0%B5%D0%BB%D1%8F%D1%82%D0%B8%D0%B2%D0%B8%D1%81%D1%82%D1%81%D0%BA%D0%B8_%D1%80%D0%B0%D0%B2%D0%BD%D0%BE%D1%83%D1%81%D0%BA%D0%BE%D1%80%D0%B5%D0%BD%D0%BD%D0%BE%D0%B5_%D0%B4%D0%B2%D0%B8%D0%B6%D0%B5%D0%BD%D0%B8%D0%B5
Аноним 08/02/15 Вск 15:57:52 #82 №237553 
>>237542
>>237545
Я же не про формулы, а как на самом деле будет. То есть какие явления эти формулы описывают. Хотя утверждается что внутри корабля движение не будет заметно, но чем внутри отличается от снаружи? Кто такой внешний наблюдатель? Можно же любой атом всех тел эксперимента считать внешним наблюдателем для всех остальных? То есть части корабля просто не разойдутся далеко, а снаряд выпущенный с носа не улетит от пушки быстро какую бы энергию не затрачивали на это?
Аноним 08/02/15 Вск 16:14:35 #83 №237555 
>>237553
>как на самом деле будет
На самом деле будет так, как описано в формулах, лол.
Аноним 08/02/15 Вск 16:36:19 #84 №237559 
>>237553
>То есть какие явления эти формулы описывают.

Описывают, по сути, устройство геометрии пространства-времени в СТО, называемого пространством-временем Минковского. Физ. смысл состоит в том, что временные и пространственные масштабы взаимосвязаны друг с другом, и эта взаимосвязь проявляется в различии измеренных длин и времен в разных ИСО, движущихся с большими относительными скоростями.

>чем внутри отличается от снаружи?
> Кто такой внешний наблюдатель?

"Внешний наблюдатель" в данном контексте - это тот, кто сидит снаружи в условно неподвижной ИСО и взирает на проносящиеся мимо него хреновины.

>То есть части корабля просто не разойдутся далеко, а снаряд выпущенный с носа не улетит от пушки быстро какую бы энергию не затрачивали на это?

С точки зрения условно неподвижного наблюдателя - да, разойдутся слабее, т.к. с его точки зрения масштабы в движущейся системе сокращаются.
Аноним 08/02/15 Вск 20:54:03 #85 №237613 
>>236779
Группа такая. Батя мой в душе всегда поет.
Аноним 08/02/15 Вск 23:05:33 #86 №237627 
14234259330060.jpg
>>237559
>С точки зрения условно неподвижного наблюдателя - да, разойдутся слабее, т.к. с его точки зрения масштабы в движущейся системе сокращаются
Мля, я про это забыл. А с точки зрения астронавтов в корабле получается ничего не изменится. То есть для них пространство между частями корабля растягивается как и время? А если они одинаково затормозят сожмется обратно и части корабля приблизятся и остановятся почти рядом как видел сторонний наблюдатель?

И еще хотел сказать, сайентачеры спасибо за ваши ответы :3
Аноним 09/02/15 Пнд 00:19:31 #87 №237643 
>>237627
>А с точки зрения астронавтов в корабле получается ничего не изменится
Да.
>То есть для них пространство между частями корабля растягивается как и время?
Время на космолете замедляется, масштабы сокращаются. Но только с точки зрения условно неподвижного наблюдателя.
>А если они одинаково затормозят сожмется обратно и части корабля приблизятся и остановятся почти рядом как видел сторонний наблюдатель?
Если космолет и выброшенные из него объекты затормозят - в процессе торможения это уже не будет инерциальная система отсчета. То, насколько разнесенными окажутся части по окончании торможения - зависит от их скоростей и режима торможения. Но, в грубом приближении, неподвижный наблюдатель при наблюдении торможения какой-то системы тел увидит растяжение масштабов между этими телами (т.к. изначально на высокой скорости они для него выглядели сжатыми).
Аноним 09/02/15 Пнд 10:59:09 #88 №237730 
>>237627
> сайентачеры спасибо
>ТО
Нахуй тебе это говно сдалось?
Аноним 09/02/15 Пнд 15:11:44 #89 №237784 
14234839046270.jpg
>>237483
то есть всё предопределено? "Искры жизни" не существуют, а мы - биокомпьютеры?
Аноним 09/02/15 Пнд 15:22:21 #90 №237791 
>>237784
>то есть всё предопределено?
Да.
Недавно Кукушкин и Казанцева отвечали на вопросы по- поводу мозга и вот там тоже был этот вопрос.

В: Я нахожусь в покое, затем осознанно, скажем, поднимаю руку. В этот момент в моем мозгу активируются какие-то зоны и посылают сигнал мышцам. Так я себе это представляю. Вопрос — что активирует эти зоны мозга? Рождается некий электрический импульс, но откуда?
+В: С точки зрения нейропсихологов поведение человека детерминировано или индетерминировано?

Коля: Мозг никогда не находится в покое. В нём всегда что-то происходит. От органов чувств идут сигналы об окружающей среде, те анализируются ассоциативной корой, компилируются и сравниваются с данными из памяти, рассылаются по проложенным дорожкам эмоций, ощущений и словесных категорий, возвращаются к центрам анализа внешних сигналов, уточняют их обработку, летят к центрам мотивации, где сходятся с сигналами от внутренних органов, активируют подходящую программу действий, которая в нужном порядке активирует моторные зоны, сигналы из которой спускаются по спинному мозгу и нервам к мышцам рук. Это и есть мыслительный процесс: самоподдерживающаяся каша нейронных сигналов. Помимо движений рук, каким-то образом эта каша создаёт у нас ощущение собственного «я», принимающего решение о поднятии руки.

Другой вопрос состоит в том, есть ли у нас контроль над происходящим, то есть свобода воли. Если честно, то кроме метафизически и религиозно направленных философов, в это не верит никто. Я считаю, что наше «я» — такая же часть перетекающей самовозбуждающейся каши нейронных сигналов, как и всё остальное. У каждой мысли есть молекулярно-клеточная причина. Другое дело, что эти причины настолько невообразимо, непознаваемо сложны, что с нашей точки зрения нет никакой разницы, вызвана ли мысль взаимодействиями молекул или какой-то абстрактной «волей». Хотя понимание собственной молекулярности, на мой взгляд, сильно помогает жить.
Короче, если подводить итог: импульс рождается из других импульсов. Это постоянное порождение одних импульсов другими и составляет наше сознание.

https://vk.com/page-74404187_49622472
Аноним 09/02/15 Пнд 17:32:44 #91 №237806 
14234923643710.jpg
>>237784
Некорректно называть людей биокомпьютерами. У людей даже простого математического сопроцессора нет, они двузначное число на двузначное умножить не могут в уме.
Аноним 09/02/15 Пнд 17:54:21 #92 №237815 
14234936619120.jpg
>>237791
>свобода воли. Если честно, то кроме метафизически и религиозно направленных философов, в это не верит никто
Коту Шредингера это чето не помогло.
Аноним 09/02/15 Пнд 23:16:57 #93 №237901 
У меня организационный вопрос, вот вы парни сколько минут в день отдыхаете, так на вскидочку.
Аноним 10/02/15 Втр 09:28:36 #94 №237962 
>>237815
Что сказать- то хотел?
Аноним 10/02/15 Втр 14:40:38 #95 №238094 
>>237962
Какой детерминизм если поведение частиц рэндомное?
Аноним 10/02/15 Втр 14:53:20 #96 №238096 
>>238094
>поведение частиц
>поведение человека.
Лол. Мозг слишком большой и горячий объект для того чтобы там как- то квантовая механика работала для мозга в целом. Все это "рэндомное" поведение частиц усредняется и дает на выходе банальный и так не любимый религиозными маньками детерминизм.
Аноним 10/02/15 Втр 15:20:11 #97 №238099 
>>238096
>Мозг слишком большой и горячий объект для того чтобы там как- то квантовая механика работала для мозга в целом
Ну вот лягушка может видеть отдельные фотоны, а человек сигнал от фотоумножителя, или реакцию лягушки на фотон, и его мозг в целом среагирует. Шах и мат детерминист.
Аноним 10/02/15 Втр 16:07:58 #98 №238107 
>>238094
...иии?
Поведение частиц вообще- то не рэндомное, а вероятностное, да и то с некоторыми оговорками.
Так что сказать- то хотел? Как у тебя вероятностность квантовой частицы даст рэндомное, "свободное" поведение мозга?
>>238099
Лол.
Аноним 10/02/15 Втр 16:45:27 #99 №238115 
Можно ли, зная строение атома, количество протонов-электронов, орбиты, вот это всё, предсказать цвет вещества, механические свойства, электрическое сопротивление, теплопроводность и тд?
Аноним 10/02/15 Втр 16:47:00 #100 №238116 
>>238115
Да.
Аноним 10/02/15 Втр 17:18:17 #101 №238120 
>>238107
>Как у тебя вероятностность квантовой частицы даст рэндомное, "свободное" поведение мозга?
Что не ясно? >>238099
Одна частица может повлиять на поведение. Алсо почитай про космические ливни, что может зделать одна частица. Или например та которая уебала фобос-грунт повлияла на многих людей.
Аноним 10/02/15 Втр 17:27:05 #102 №238121 
>>238120
>Что не ясно?
Где тут свободное не детерминированное поведение человека.
Аноним 10/02/15 Втр 17:34:04 #103 №238123 
>>238120
>уебала фобос-грунт повлияла на многих людей
>частица
>уебала фобос-грунт
Два по 15 этому патриоту, лол.
Аноним 10/02/15 Втр 18:49:00 #104 №238141 
>>238121
А где тут детерминированное?
Олсо у тебя в мозгу тупо может какая-нить молекула квантово вздочнуть, нейрон возбудится, обзвонит все 1000 нейронов с которыми связан, и хуяк ты вспомнил что-то и принял решение хотя ничто не предвещало.
Аноним 10/02/15 Втр 18:59:22 #105 №238143 
>>238141
>где тут детерминированное
Там же, где и было раньше. Поведение человека как определялось "биологией и химией" так и определяется дальше, без всякой "свободной воли". И с какого хуя ты вдруг решил, что внешнее и никак не управляемое человеком "космическая частица ебнула" доказывает наличие у человека свободной воли, мне по- прежнему не понятно. Это примерно как говорить- голубь случайно насрал человеку на голову, следовательно у человека есть свободная воля.
>в мозгу тупо может какая-нить молекула квантово вздочнуть, нейрон возбудится, обзвонит все 1000 нейронов с которыми связан
Нет, не может.
Аноним 10/02/15 Втр 20:24:27 #106 №238154 
14235890676660.png
>>238116
> Да.
Ну давай, распиши, как ты будешь действовать, чтобы узнать теплопроводность пикрелейта? Ессно не по таблицам готовым, а с нуля. Название зачеркнул чтобы не смущать тебя.
Аноним 10/02/15 Втр 20:38:29 #107 №238157 
>>238143
>Нет, не может.
Да ты охуел.
>Поведение человека как определялось "биологией и химией" так и определяется дальше, без всякой "свободной воли".
Лол, это вообще о чем?
Аноним 10/02/15 Втр 20:45:12 #108 №238159 
Неважно, детерминизм у нас или интедерминизм - они в равной степени нивелируют метафизическую свободу воли. В первом случае - потому что тогда все деяния человека заранее предопределены, свободный выбор иллюзорен; во втором - потому что тогда деяния человека есть игра неподконтрольного ему слепого случая, и свободный выбор опять иллюзорен. Это так называемый стандартный аргумент против свободы воли (http://www.informationphilosopher.com/freedom/standard_argument.html)

Максимум, что тут можно сделать - сказать, что в контексте человеческой практики и этики важно не то, чем на самом деле предопределены наши действия на низкоуровневом левеле, а наше индивидуальное ощущение пространства выбора и общественных последствия тех действий, которые мы согласно этому ощущению присваиваем себе как "выбранные".
Аноним 10/02/15 Втр 20:47:24 #109 №238160 
>>238159
(Алсо, по ссылке можно охуеть от того, насколько подробно эти вещи уже давным-давно обсуждены, и следовательно, насколько бесполезным и дилетантским являются беспруфные кукареканья двачеров, изобретающих кособокие философские велосипеды).
Почему прочность указывается в кг НА ПЛОЩАДЬ? Аноним 10/02/15 Втр 20:50:36 #110 №238161 
Сверху свисает метровый трос, на нём подвешено ведро.
Макс. нагрузка на трос – 5кг/метр, т.е. он порвётся при весе ведра в 5кг, а если укоротить его в 2 раза – разве порвётся при 2.5кг?
Аноним 10/02/15 Втр 20:59:46 #111 №238162 
14235911868970.jpg
Почему на двачах все пиздят, что они студентота по 20-22, когда большинство даже в полное среднее образование не может?
Аноним 10/02/15 Втр 22:08:15 #112 №238184 
>>238159
>во втором - потому что тогда деяния человека есть игра неподконтрольного ему слепого случая, и свободный выбор опять иллюзорен
Ну определи что по твоему значит "свободный". Алсо разьедини игру и реакцию на нее. А то чето больно гуманитарно.
Аноним 10/02/15 Втр 22:51:31 #113 №238198 
>>238184
>Ну определи что по твоему значит "свободный"

Увы. Все ломания копий вокруг этого понятия упираются в неформализуемое, почти инстинктивное ощущение того, что мол, именно Я а что такое "я" - тож нихуя никто четко не скажет, и никто/ничто другое, ответственен за свой волевой акт. И что это, дескать, персонально "мой" выбор, а не выбор за меня (который мог бы проистекать от жесткой детерминации или от хуйпойми каких-то случайностей, к моему чувству мотивирующего/волящего "я" относящихся не более, чем к васиному или петиному (т.е. ни к чьей воле конкретно не относящихся вовсе)).

Вот. Приведенный ранее "стандартный аргумент против свободы воли" как раз и показывает, что такое инстинктивное понимание свободы - пустое множество, филологический самообман. Нет большой разницы, за счет чего твой выбор делается - за счет ли жесткой предопределенности (и тогда ты автомат), либо за счет случайностей (по определению беспричинных, т.е. не проистекающих ни из чьей самости). В любом случае ололо-самость оказывается ни при делах.

Правда, некоторые под свободой мыслят, например, непредсказуемость. Тогда, конечно, "свободными" становятся любые случайности. Другой вопрос - а толку-то от такого определения свободы. Уйдя от неподконтрольного твоей воле удушливого фатализма, попадаешь в мир, где за тебя все решают неподконтрольные же твоей воле случайности.
Аноним 10/02/15 Втр 23:10:33 #114 №238209 
>>238198
Да, ну и тут, конечно, можно было бы хитровыебнуться и постулировать пресловутую causa sui причину себя: мол, причина моих действий лежит ни во внешней детерминации, ни в случайностях, а в ололо-способности субъекта с нуля генерить новую причинно-следственную цепь.

Это успокаивающий трюк, но если разобраться подробнее, то и тут возникают проблемы. Ну хорошо, ну "сгенерилась" типа цепь событий субъектом; но если она была хоть чем-то обусловлена -- пусть даже каким-то глубоко внутренними, неотрефлексированными мотивациями, однозначно ее предопределившими так а не иначе -- значит, всё равно была предопределенность заранее, просто не фикисируемая осознанной волей (последняя была бы уже апостериорна, т.е. не свободна). А если же эта цепь не была обусловлена ничем - тогда это ничем не отличается от случайности, и слова "сгенерилась субъектом" становятся пустыми.
Аноним 10/02/15 Втр 23:15:13 #115 №238220 
>>238198
>Тогда, конечно, "свободными" становятся любые случайности. Другой вопрос - а толку-то от такого определения свободы.
Свобода и свобода воли это разные вещи. Свободным является что-либо безусловное, независимое. Свобода воли же - это просто понятие в котором уже заключено внутренние противоречие, поэтому нечто такое как свобода воли вообще не может существовать.
Аноним 10/02/15 Втр 23:16:59 #116 №238222 
>>238220
>поэтому нечто такое как свобода воли вообще не может существовать.

Ну тем более.
Аноним 10/02/15 Втр 23:40:00 #117 №238249 
>>238198
Околесица это. Про детерминизм еще понятно, типа все связано жестко, но это же вообще не имеет никакой логики. Я это вполне конкретный механизм который может много чего делать или не делать. Тем более детерминизма нет и можно действовать по обстоятельствам, ничего вообще не предопределено. А то у тебя сейчас выйдет что камень свободен потому что ничего не делает.
>>238220
>Свободным является что-либо безусловное, независимое
Што?
Аноним 11/02/15 Срд 00:12:48 #118 №238275 
>>238249
>Околесица это
По ссылкоте >>238159 http://www.informationphilosopher.com/freedom/standard_argument.html не скажешь.
Аноним 11/02/15 Срд 00:14:01 #119 №238276 
>>238115
>Можно ли, зная строение атома, количество протонов-электронов, орбиты, вот это всё, предсказать цвет вещества, механические свойства, электрическое сопротивление, теплопроводность и тд?
Бамп вопросу
Аноним 11/02/15 Срд 00:20:20 #120 №238279 
>>238249
>Тем более детерминизма нет и можно действовать по обстоятельствам, ничего вообще не предопределено.

Допустим, но к "свободе воли" это не имеет отношения. Детерминизм, индетерминизм - один хрен. В конечном итоге выбор индивида решают либо жесткая причинно-следственная связь (фатализм), либо случайности. Места для пафоса решаю только "я", а не что-то кто-то "за меня" не остается.

А так, если тебе спокойно именно от индетрминизма - ну и хорошо. Это уже вопрос психологии, кому что приемлемее: мириться, что ты автомат, или мириться, что в тебе клокочет море случайностей.
Аноним 11/02/15 Срд 00:29:09 #121 №238285 
14236037494090.jpg
>>238276
Этот дядька вырьет твою кровь в аду.
Аноним 11/02/15 Срд 00:29:35 #122 №238286 
>>238285
выпьет
Аноним 11/02/15 Срд 00:43:49 #123 №238287 
>>238275
Там много букв лень разбирать. Вкраце о чем там?
>>238279
>жесткая причинно-следственная связь (фатализм)
Это детерменизм обоссаный квантовыми механиками.
>либо случайности
Случайности ничего не решают, это я решаю.
Это все напоминает какие-то апории.
Аноним 11/02/15 Срд 01:01:40 #124 №238289 
>>238287
>Это детерменизм обоссаный квантовыми механиками.
Да это-то понятно (хотя на самом деле не такой уж и обоссанный, единственное, что реально обоссано - это запрет на локальные теории скрытых параметров из-за неравенств Белла, что не запрещает выдумывать интерпретации квантмеха за пределами копенгагенской, с нелокальными скрытыми параметрами, где будет прошит скрытый детерминизм, типа теории Бома, насколько я помню. Но это здесь не суть важно, допустим, детерминизма нет и в помине.)

>Случайности ничего не решают, это я решаю.
Ну вот, видишь, вот эта апелляция к Я решаю звучит внушительно, но что именно за ней скрывается такого, что не сводилось бы случайности/детерминизму?
Аноним 11/02/15 Срд 01:33:03 #125 №238292 
>>238287
>Там много букв лень разбирать. Вкраце о чем там?

Да о том же примерно, что и здесь.
Как понять прочность троса? Аноним 11/02/15 Срд 07:04:36 #126 №238299 
Сверху свисает метровый трос, на нём подвешено ведро.
Макс. нагрузка на трос – 5кг/метр, т.е. он порвётся при весе ведра в 5кг, а если укоротить его в 2 раза – разве порвётся при 2.5кг?
Аноним 11/02/15 Срд 07:10:23 #127 №238301 
>>238299
Откуда ты взял /метр? Так и говорят - трос на 5 кг.
Аноним 11/02/15 Срд 07:14:12 #128 №238303 
>>238301
Ну на квадратный мм (в спецификации так). В ширину он 1мм, при укорачивании в 2 раза квадратных мм будет в 2 раза меньше.
Аноним 11/02/15 Срд 07:17:39 #129 №238304 
>>238303
Это сечение, няша.
Аноним 11/02/15 Срд 07:53:30 #130 №238309 
>>238304
И? Сечение 1мм, при укорачивании в 2 раза в тросе останется в 2 раза квадратных мм.
Аноним 11/02/15 Срд 07:57:26 #131 №238310 
меньше
Аноним 11/02/15 Срд 08:01:18 #132 №238312 
>>238157
О том, что у человека нет свободной воли.
>>238289
Есть идея, вроде восходящая чуть ли не к Бору, что смягчение классического детерминизма в квантовой механике на самом деле кажущееся и возникает из- за "языковых" проблем. Т.е. из- за описания квантовых объектов с помощью по сути классических понятий- импульса, координаты итп. И поэтому квантовая механика на самом деле вполне жестко-детерминистична, а все эти вероятности- просто баг описания, возникающий из- за такого себе натягивания совы на глобус, из- за попытки описывать КМ в чуждой ей понятийной схеме.
Аноним 11/02/15 Срд 08:03:57 #133 №238313 
>>238310
При укорачивании троса у него не умешьшится площадь сечения. Если ты распустишь трос на канатики, то каждый из них сможет поднять те же 5 кг на мм2, но сечение канатиков будет меньше, и вес они будут поднимать меньший.
Аноним 11/02/15 Срд 08:15:21 #134 №238315 
>>238313
Т.е. длина без разницы? Спасибо.
Аноним 11/02/15 Срд 08:21:34 #135 №238316 
>>238313
А разве сечение не должны указывать в мм? Т.е. должны тогда писать 5кг на 1мм сечение? А не на квадратный мм..
Аноним 11/02/15 Срд 08:25:47 #136 №238317 
Ах да, это ж не кубические мм, понял.
Аноним 11/02/15 Срд 08:33:12 #137 №238318 

> Можно ли, зная строение атома, количество протонов-электронов, орбиты, вот это всё, предсказать цвет вещества, механические свойства, электрическое сопротивление, теплопроводность и тд?
АП
Аноним 11/02/15 Срд 09:22:50 #138 №238320 
>>238289
>но что именно за ней скрывается такого, что не сводилось бы случайности/детерминизму?
У тебя есть какие-то другие понятия? Нет? Тогда о чем ты вообще пишешь?
>>238312
Восходящая к багету эйнштейна. Скажи еще что атомы распадаются не рэндомно.
Аноним 11/02/15 Срд 09:24:22 #139 №238322 
>>238320
>атомы распадаются не рэндомно
ничто не случайно.
Аноним 11/02/15 Срд 09:26:04 #140 №238325 
>>238322
Как боженька велел?
Аноним 11/02/15 Срд 09:28:21 #141 №238326 
>>238325
велит он слушающим.
остальным он управляет.
Аноним 11/02/15 Срд 11:05:54 #142 №238340 
>>238320
>Восходящая к багету эйнштейна
Нет
>Скажи еще что атомы распадаются не рэндомно
Нет, не рэндомно.
Попустись уже, нет никакой свободы воли в фундаментальном смысле и квантовая механика никак в этом не поможет.
Аноним 11/02/15 Срд 14:21:21 #143 №238388 
>>238320
>У тебя есть какие-то другие понятия? Нет? Тогда о чем ты вообще пишешь?
Так о том и речь, что никакая апелляция к "я решаю" не поможет, потому что других понятий, кроме детерминизма/случайности, в данном случае нет. А ни то, ни другое не оставляет место для "Я решаю".
Аноним 11/02/15 Срд 15:26:00 #144 №238433 
14236575606070.png
>>238340
>Нет, не рэндомно
Где это написано? Как вычислить когда данный атом распадется?
>>238388
Но решаю то я, лол. Я строю себе детерминизм сам, вопреки рэндомным случаям. У меня есть свобода строить детерминизм который мне больше нравится. А если нет даже понятия об альтернативе, то теория нефальсифицируема, об чем речь вообще?
Аноним 11/02/15 Срд 15:45:52 #145 №238453 
>>238433
>Но решаю то я
Т.е. твое "я" - это что-то такое, что стоит над оппозицией "детерминизм/случайность"? Какая-то третья инстанция, в которой события генерятся ни детерминистично, ни случайно?

>А если нет даже понятия об альтернативе, то теория нефальсифицируема
Какое конкретно утверждение ты считаешь нефальсифицируемым?
Аноним 11/02/15 Срд 16:09:37 #146 №238470 
>>238433
Ох лол. Тебе уже выше по треду написали, что не надо путать "рэндомное" и "вероятностное". Распад атомов- вероятностный процесс, а не случайный.
И даже если вероятностное поведение частиц вдруг внезапно с какого- то перепугу, за счет какой- то неведомой науке магии сможет сделать твое поведение менее детерминистичным, то даже тогда "решать" будут квантовые законы, а не твоя свободная воля, лол.
Аноним 11/02/15 Срд 18:01:40 #147 №238569 
>>238453
>Какое конкретно утверждение ты считаешь нефальсифицируемым?
Что нет свободы воли если есть только детерминизм и случайность. Придумай волю вообще без причинных связей.
>>238470
>Распад атомов- вероятностный процесс, а не случайный.
А че случайность уже не вероятностная? Когда распадется атом, сейчас, а может никогда?
>"решать" будут квантовые законы
Чето эти законы даже каменного топора создать не смогли сами.
Аноним 11/02/15 Срд 19:26:13 #148 №238581 
>>238569
Фальсифицируемость (попперовская) - это хорошая штука, ее профиты как критерия научности тех или иных эмпирических гипотез и теорий неоспоримы. Однако ясно, что в применении к вещам, лежащим за пределами эмпирики (а именно таковы, например, метафизические конструкты) или к логическому анализу понятий (в данном случае - логическому анализу содержательности понятия свободы воли) критерий фальсифицируемости иррелевантен сути дела. Т.е. его применяй/ не применяй - суть дела не изменится.

Поясняю на примере. Если бы критерий фальсифицируемости распространялся на логический анализ понятий, тогда с тем же успехом можно было заявить, что мы не можем утверждать о несуществовании логически противоречивых понятий типа круглого квадрата (на том, дескать, основании, что такое утверждение нефильсифицируемо (неопровержимо никаким опытом); ну так ясен пень -- оно неопровержимо просто потому что опытный контрпример просто логически немыслим). Т.е. да, оно неопровержимо - но лишь как следствие внутренних логических траблов в своей структуре.

Вот в данном случае понятие свободы воли - это примерно тот же "круглый квадрат": суть ранее сказанного состояла в том, что данное понятие логически некогерентно.

Но у тебя есть шанс попытаться сделать его фальсифицируемым, если ты сможешь придумать такой опыт, в котором твоя свободв воли будет нетривиальна, и будет именно интимнейшим образом твоей (что это такое - вот тоже вопрос!), а вовсе не следствием предопределяющего всё и вся детерминизма или следствием хаоса (неподконтрольного уж вообще никому конкретно).
Аноним 11/02/15 Срд 20:18:23 #149 №238587 
>>238581
>Фальсифицируемость (попперовская) - это хорошая штука, ее профиты как критерия научности тех или иных эмпирических гипотез и теорий неоспоримы
Это просто тест на осмысленность умопостроения, в т.ч. и понятий.
>понятий типа круглого квадрата
Понятий типа квадратного квадрата. Оно не имеет смысла потому что нет круглого квадрата или еще какого-нибудь треугольного. А круглого нету потому что например углов у круга нету.
>а вовсе не следствием предопределяющего всё и вся детерминизма или следствием хаоса (неподконтрольного уж вообще никому конкретно)
Если один атом поместить в коробку не известно когда и что с котом произойдет. Если обычный таймер, то будет известно. Вот хочешь детерминизм сделай, хочешь случайность, хочешь там какой-нибудь хитрый алгоритм из детерминизмов и случайностей.
Аноним 11/02/15 Срд 21:51:34 #150 №238629 
>>238276
>>Можно ли, зная строение атома, количество протонов-электронов, орбиты, вот это всё, предсказать цвет вещества, механические свойства, электрическое сопротивление, теплопроводность и тд?
>Бамп вопросу
>
Аноним 11/02/15 Срд 23:12:18 #151 №238642 
>>238629
Ну вроде как можно, но не на 100%. Периодическая таблица же для этого, по ней гадают.
https://ru.wikipedia.org/wiki/Периодическая_система_химических_элементов
Аноним 12/02/15 Чтв 00:08:08 #152 №238662 
>>238115
Тут скорее всего эти свойства надо гадать по молекулам, а не по атомам
Аноним 12/02/15 Чтв 00:17:11 #153 №238665 
>>238115
Из перечисленных тобой свойств атома не вытекают однозначно свойства вещества. Пример - аллотропные модификации какого-нибудь элемента: атом один и тот же, а структура соединений разная. И именно она-то и оказывается ключевой. Так что правильно замечено в >>238662 - специфика физико-химических свойств возникает на уровне скорее уж молекулярных соединений (inb4 редукционисты как обычно соснуле, хотя что-то в этом есть).
Аноним 12/02/15 Чтв 00:24:21 #154 №238667 
>>238665
А еще молекулы, блядь, могут образовывать разные элементарные ячейки(ну там кубическая, тетрагональная, гексагональная и т.д.) и ассоциации - полиморфизм сложных веществ, например CaCO3 - кальцит и арагонит, у которых свойства тоже разные, блядь
>>238662-кун
Аноним 12/02/15 Чтв 01:08:30 #155 №238674 
>>236711
1) Что такое душа?
2) Эфир изначально создавался как нулевой элемент, но нулевой элемент не имеет смысла, у него нет протона, значит нет электрона, значит нет химического взаимодействия, хотя некоторые берут за нулевой элемент нейтрон, но ведь нейтрон не может участвовать в химическом взаимодействии. Кароче, нулевого элемента нету, тогда что сторонники эфира, берут за эфир? Если тот самый эфир не имеет смысла. Если это какая-то особая среда, то чём она отличается от моря рождающихся и умирающих виртуальных частиц и античастиц в вакууме?
3) Гравитационная волна - это волна пространства времени. Есть Пузырь Алькубьерре, в котором само пространство-время моет двигаться со скоростью выше скорости света. Тогда почему почти везде считается что гравитоны и гравитационные волны движутся со скоростью света? Это ведь не элементарные частицы и не материя это просто какая-то часть пространства времени которая хуй ложила на запрет превышения скорости света.
4) Если же мы все-таки доберемся до абсолютного нуля это грозит пиздецом? Ну смотрите 0 К - нет энергии, считается что мы живем в ложном вакууме с какой-то там минимальной энергией, добились 0 К, это часть пространства с 0 К забирает энергию из окружающей среды, но там уже образуется не ложный вакуум, а истинный который со скоростью света пидорашит ложный, как предсказывают ученые про нестабильный бозон Хиггса который может туннелировать в истинный вакуум и так же пидорашить ложный, в котором мы живём.
Аноним 12/02/15 Чтв 02:00:12 #156 №238682 
>Ta-180 период полураспада 8,152 часа, ядерный изомер Ta-180m1 период полураспада 7,1 умножить на 10^15 лет. https://ru.wikipedia.org/wiki/Изотопы_тантала
Это блядь что получается если взять все возможные изотопы радиоактивных элементов (уран, франций, унуноктий(118), коперниций, унунетий(119) и т.д.) от границы энергии отделения нуклона до границы энергии отделения протона(см. N-Z диаграмма ядер/нуклидов) и закидать их во всевозможные ядерные возбужденные состояния мы таки можем получить стабильное ядро? Так какого хуя этим ещё не занимаются?
Аноним 12/02/15 Чтв 06:53:49 #157 №238699 
>>238309
ты совсем наркоман штоле сука, поперечное сечение
чем тоньше тем слебее
Аноним 12/02/15 Чтв 21:21:26 #158 №238803 
14237652861930.jpg
Сосоны, из воздуха сделали посев на питательную среду. Выросло что-то. Сделали раздавленную каплю, не подкрашенную.

Пикрелейтед. Там что-нибудь есть? Или это воздух?
Аноним 12/02/15 Чтв 21:29:21 #159 №238804 
14237657612790.jpg
Анон, распиши, пожалуйста, какие темы теорвера нужно знать для решения данных задач.
Аноним 12/02/15 Чтв 22:36:57 #160 №238817 
>>238804
> диаметр вала измерили 9 раз

Блядь, они его мартышками меряли или может попугаями? человек есть мера всего

Штангелем надо мерить или микрометром, если мало точности. Охуеть нахуй.
Аноним 13/02/15 Птн 03:58:29 #161 №238884 
>>238817
Ты же школьник, верно?
Аноним 13/02/15 Птн 08:13:58 #162 №238901 
http://lib.karraba.net/wp-content/uploads/2014/09/Files/Files2/ychislitelnaya_lineinaya_algebra_s_primerami_na_matlab.pdf
хорошая книга? стоит ли заказать или копипаста и фуфло?
Аноним 13/02/15 Птн 10:31:25 #163 №238914 
>>238901
>стоит ли заказать
Что заказать? Ты же и так на нее ссылку дал.
Аноним 13/02/15 Птн 11:39:30 #164 №238932 
>>238914
ахаха да. не могу с экрана.
Аноним 13/02/15 Птн 19:30:23 #165 №239066 
>>238665
>Из перечисленных тобой свойств атома не вытекают однозначно свойства вещества. Пример - аллотропные модификации какого-нибудь элемента: атом один и тот же, а структура соединений разная. И именно она-то и оказывается ключевой. Так что правильно замечено в >>238662 - специфика физико-химических свойств возникает на уровне скорее уж молекулярных соединений (inb4 редукционисты как обычно соснуле, хотя что-то в этом есть).
Океееей. Можно ли, зная строение атома, узнать, в молекулу какой конфигурации он выстроится при тех или иных условиях и в какую кристаллическую решетку, а уже по ней - узнать описанные мною макросвойства вещества?
Аноним 13/02/15 Птн 19:38:06 #166 №239073 
>>238884
>Ты же школьник, верно?
Второй курс тащемта
Аноним 13/02/15 Птн 20:23:29 #167 №239074 
>>239066
Вполне.

С молекулами еще более-менее, собственно, вся химия этим занимается.

Вот с кристаллическими решетками сложнее - куда больше рассчетов.

Ну и между кристаллической решеткой и макро-веществом тоже есть немало промежуточных уровней. Форма и дефекты кристаллов, размеры зерен, прочее прочее...

Прямое моделирование требует неподъемно большой вычислительной мощности, поэтому приходится придумывать упрощенные методы, проверяя их валидность экспериментом.
Аноним 13/02/15 Птн 21:56:55 #168 №239091 
В розетке 2 фазы, одна из них – 0, если я с помощью трансформатора или т.п. пущу 0 по проводу глубоко в землю – может из земли потечь ток? Если нет, то почему?
Аноним 13/02/15 Птн 22:28:30 #169 №239095 
>>239091
Лолщто? Схему нарисуй, словами ты хуйню написал. Впрочем ток из земли не потечет в любом случае.
мимоэлектрик
Аноним 13/02/15 Птн 23:09:09 #170 №239098 
Прив всем.
Давненько не заходил в саентач, но возник один вопрос на котором я чёт стопорнулся. На чём основана вся "физикохимия" изменения свойств от света? Что происходит с материалами? Почему полимеры от солнца рассыпаются, краски выцветают (а другие не выцветают бляд), или почему фоторезист твердеет и ещё меняет цвет "наоборот" (с почти прозрачного до фиолетового).
http://chip-cont.ru/Details.aspx?ProductID=IDP0018913
есть клин Штоуффера, просто пластинка с разным затемнением, котроая дозирует количество энергии фотонов поглощённой телом.
http://www.stouffer.net/Tradition.htm

Но щито происходит с самими телами?
Аноним 13/02/15 Птн 23:43:18 #171 №239103 
>>239098
Фотон с достаточно большой энергией (обычно УФ) хуячит по молекуле, она переходит в возбужденное состояние и вступает в какую-нибудь реакцию.

Растения используют этот эффект для дела, там ловят фотоны на хлорофилл и используют его для полезных синтезов, а всем остальным он мешает.

Краски выцветают из-за разрушения молекул красителей, пластики становятся ломкими, потому что рвутся цепочки полимеров.
Аноним 14/02/15 Суб 00:04:19 #172 №239105 
Если ли какие-то заметки (может биография какого-то ученого) по организации рабочего времени научного работника-экспериментатора? С подсказками по ведению рабочих записей, распределению времени и приоритетов и т.д.
Аноним 14/02/15 Суб 03:09:40 #173 №239143 
>>239098
Касательно фоторезиста. Просто изменяются свойства кристаллической решетки материала, под воздействием энергии света.
Аноним 14/02/15 Суб 03:15:46 #174 №239145 
>>239095
Хах, вспомнилось почему то, как в 10 классе наш физик любил тралить всех ТП-гуманитариев в нашем классе вопросом: почему ток из розетки не вытекает на пол?
Аноним 14/02/15 Суб 03:48:55 #175 №239147 
Почему свет приламывается если из одной среды переходит в другую?Например, из вакуума в прозрачное вещество.Я понимаю, что скорость света будет разной в зависимости от среды.
Аноним 14/02/15 Суб 04:27:16 #176 №239157 
>>239147
Ну ты сам ответил на свой вопрос. Скорость света в разных средах разная. Соответственно и фазовые скорости волн тоже будут разными. Из-за этой разности в фазовых скоростях волны ты и наблюдаешь эффект преломления.
Аноним 14/02/15 Суб 04:27:34 #177 №239158 
как определить кажущийся размер предмета на определенном расстоянии? Ну смотришь ты сквозь стекло на некий квадрат метр на метр. Расстояние минимально, квадрат занимает все поле зрения через стекло. Сколько будет занимать квадрат на расстоянии 10, 100, 1000 м? как рассчитать?
Аноним 14/02/15 Суб 04:41:14 #178 №239160 
>>239157
Анончик, можешь, пожалуйста, объяснить подробнее про разность фазовых скоростей волн? Как 7 класснику.
Аноним 14/02/15 Суб 05:00:05 #179 №239164 
>>239160
Да ты ахуел такое просить, если тебе интересно поподробней, с формулами и рисунками, то иди читай учебник!
Аноним 14/02/15 Суб 05:29:37 #180 №239168 
>>239158
Может, что-то с угловыми размерами?
Аноним 14/02/15 Суб 05:31:48 #181 №239169 
>>239164
Блять , я читал это гавно, но нихуя не понял, я тупое быдло. Будь няшкой , анонимус.
Аноним 14/02/15 Суб 09:47:58 #182 №239189 
>>239095
>Впрочем ток из земли не потечет в любом случае.
Но в инете же есть схемы зарядников которые получают ток из трубы в земле – неправды?
Аноним 14/02/15 Суб 11:32:13 #183 №239201 
14239027336570.jpg
>>238804
Бамп. Хотя бы кратко. Хочу решить сам, но штудировать учебник целиком некогда.
Аноним 14/02/15 Суб 13:50:26 #184 №239234 
Почему ток из розетки не вытекает на пол?!
Аноним 14/02/15 Суб 14:06:20 #185 №239236 
>>239234
сопротивление воздуха между полом и розеткой большое.
Аноним 14/02/15 Суб 14:45:10 #186 №239251 
>>238682
>>238674
Бамп
Аноним 14/02/15 Суб 14:51:29 #187 №239253 
14239146891460.jpg
Друзья, подскажите, сколько джоулей электрической энергии необходимо для сокращения сердечной мышцы (не для реанимации, а для одного сокращения)
Аноним 14/02/15 Суб 14:55:25 #188 №239258 
>>239253
Столько же, сколько для сокращения, мань. Реанимация - это и есть принудительное сокращение.
Если вводить электроды к сердцу, то понты. А при разряде снаружи основная часть уходит на поджаривание кожи.
Аноним 14/02/15 Суб 15:02:33 #189 №239266 
>>239258
Ну и сколько нужно, если без поджаривания кожи?
Аноним 14/02/15 Суб 15:13:10 #190 №239268 
>>239258
>>239266
Если использовать меньше 100 Дж, что-нибудь получится?
Аноним 14/02/15 Суб 18:05:42 #191 №239302 
Почему у фотона нету массы? Как так получилось, что все элементарные частицы её имеют, а фотон нет?
Аноним 14/02/15 Суб 18:08:24 #192 №239303 
>>239302
прост)))
Аноним 14/02/15 Суб 18:50:26 #193 №239310 
>>239302
Все калибровочные бозоны не имеют
Аноним 14/02/15 Суб 18:55:29 #194 №239312 
>>239310
У которых группа не нарушена
Аноним 14/02/15 Суб 19:07:31 #195 №239314 
Аноны, подскажите, будьте добры. Нужно построить график рассеивания величин двух объектов, каждый по два признака. Грубо говоря - члены негров и члены белых. Два признака - длина и диаметр. Нашел в Statistica двумерные график рассеивания, но он строит только на одну пару признаков, или на несколько несвязанных. А мне надо именно две пары связанных признаков. Может я в глаза ебусь и не вижу там опции на две пары?
Химик-кунам Аноним 14/02/15 Суб 20:18:15 #196 №239331 
Как можно извлечь нитроглицерин из таблетки?
Аноним 14/02/15 Суб 21:02:23 #197 №239347 
>>239331
Его там мало, тупо пронитровать глицерин не вариант?
Электричество из земли Аноним 15/02/15 Вск 08:27:18 #198 №239420 
>>239189
Аноним 15/02/15 Вск 10:01:20 #199 №239429 
>>239331
Товарищ майор?
Аноним 15/02/15 Вск 15:12:17 #200 №239508 
14240023379920.png
Кто это такой? Как зовут? Где живет? Где работает?
Аноним 15/02/15 Вск 15:22:24 #201 №239510 
это munin.
Аноним 15/02/15 Вск 19:01:45 #202 №239537 
>>239508
А что, бугурт от него уже нестерпим?
Аноним 15/02/15 Вск 19:27:33 #203 №239541 
>>239537
Ты прозорлив. Этот тип представляет собой уникальный эталон образованческого мировоззрения, прям из палаты мер и весов, даже немного стыдно жить с ним на одной планете. Но уникумов надо знать в лицо. Так кто он?
Аноним 15/02/15 Вск 19:43:15 #204 №239547 
14240185953170.jpg
>>239541
Аноним 16/02/15 Пнд 02:50:59 #205 №239613 
Во всех атомах есть электроны, которые быстро двигаются, и ядра, которые тоже двигаются. И еще они(электроны и ядра) имеют электрический заряд. Значит они порождают магнитное поле. И в молекулах электроны, которые связывают атомы, тоже быстро двигаются от одного атома к другому. Но почему тогда магнитным полем обладают только некоторые вещества(магниты)? И еще: почему есть тела с электрическим полем? Ведь во вселенной все движется и тела с электрическим полем должны порождать только магнитное поле, за исключением если эти тела двигаются с абсолютно одинаковой скоростью, что создаёт иллюзия покоя относительно друг друга, но блядь это совсем редкость.
Аноним 16/02/15 Пнд 08:36:40 #206 №239626 
>>239613
Нахуя спрашивать о том, что можно найти даже в школьном учебнике по физике за 8 класс? Ты в каком классе, сынок?
>почему тогда магнитным полем обладают только некоторые вещества(магниты)?
Потому что в большинстве веществ атомы и и создаваемое ими магнитное поле ориентированы хаотично, за счет этого поля отдельных атомов "гасят" друг друга. В магнитах же они ориентированны упорядоченно, поэтому у магнита есть магнитное поле.
И да, магнитное поле в веществе появляется из- за движения электронов. Ядра, по сравнению с электронами почти неподвижны.
>почему есть тела с электрическим полем? Ведь во вселенной все движется и тела с электрическим полем должны порождать только магнитное поле, за исключением если эти тела двигаются с абсолютно одинаковой скоростью, что создаёт иллюзия покоя относительно друг друга, но блядь это совсем редкость.
А почему бы им не быть? У неподвижного заряженного тела есть только электростатическое тело, если ты начнешь его "шевелить", то от него побежит электромагнитная волна- изменяющееся электрическое будет порождать изменяющееся магнитное, изменяющееся магнитное- изменяющееся электрическое итд. Просто в реале ты не сможешь шевелить заряженное тело с достаточной частотой, чтобы "электродинамические" эффекты как- то явно проявились.
Аноним 16/02/15 Пнд 08:38:37 #207 №239627 
>>239626
>У неподвижного заряженного тела есть только электростатическое поле
Фикс.
Аноним 16/02/15 Пнд 09:22:09 #208 №239631 
Объясните проблему соизмеримости. Каким образом диагональ какого-либо квадрата и его сторона не имеют общей меры?
Аноним 16/02/15 Пнд 09:43:29 #209 №239633 
>>239314
Ну бамп же. Или может знаете более пригодную программу для моих целей.
Аноним 16/02/15 Пнд 09:57:22 #210 №239635 
>>239189
> Но в инете же есть схемы зарядников которые получают ток из трубы в земле – неправды?
Ну-ка доставь?
Аноним 16/02/15 Пнд 10:33:19 #211 №239637 
>>239633
короче матлаб блять учи. там щас и статистика и маткад и вся хуета.

мимодвоечник по матлабу
Аноним 16/02/15 Пнд 15:39:24 #212 №239681 
14240903643740.jpg
>>238804
Ещё раз. Неужели никто не знает?
Аноним 16/02/15 Пнд 20:26:50 #213 №239718 
>>239631
Мера тут ни при чем, под несоизмеримостью греки понимали то, что отношение диагонали квадрата к стороне внезапно не рационально, и охуевали от того, как же так выходит - мол, нельзя конечным числом откладываний долей отмерить корень из двух.
Аноним 17/02/15 Втр 00:30:18 #214 №239760 
14241222184640.jpg
>>239253
Механическая работа одного сердечного сокращения в покое – около 1 Дж. А для запуска сокращения, если электрод введен непосредственно в проводящую систему сердца, энергия нужна ничтожная – мощность кардиостимуляторов составляет всего 10 мкВт, поэтому заряда их батарей хватает на многие годы.
Аноним 17/02/15 Втр 08:12:38 #215 №239778 
И так вот какой вопрос от гуманитария. Бесконечность возможна?
Аноним 17/02/15 Втр 08:49:50 #216 №239781 
>>239778

Как потенциальная - да.

Как актуальная - хз:
1. В обычном (неконструктистском) мат. анализе и канторовской теории множеств - актуальная беск. возможна как логическая абстракция, дающая возможность говорить о континууумах и счетных множествах как о чем-то завершенном и данном. Интуиниционисты/конструктивисты налагают на такие абстракции запрет.
2. В материальной реальности - вообще хз. Если бы материя не имела пределов делимости (такое в принципе логически возможно, если бы не сука кварки и электроны), тогда бы, наверное, можно было говорить о континууме материи как об актуальной бесконечности материальных "точек". (Аналогично - с пространством-временем.) Точно так же если бы Вселенная не имела начала в прошлом (Большой Взрыв ололо), тогда бы можно было говорить об актуальной бесконечности всего множества предшествующих событий. При том, что насчет Большого Взрыва как абсолютного начала мнения расходятся - это мог быть лишь один из фазовых переходов в Мультивселенной Ну и наконец, сингулярности (космологические и чернодырные) - тоже вопрос, насколько там корректно говорить о несглаженной квантовыми флуктуациями истинно бесконечной плотности/кривизне (скорее всего, нельзя) и т.д.

Во всяком случае, актуальная бесконечность несхватываема конечными мозгами во всей полноте одновременного различения/индивидуации всех своих элементов. Максимум, что можно - это представить себе что-то типа ололо-отрезка прямой и сказать "вот, я типа представил континуум!" - хотя ясен хуй, что представил ты не актуальную бесконечность как таковую, а некую ее ограниченную полуинтуитивную модель восприятия, где элементы слиты воедино и не различены внутренним взором как индивидуализированные элементы бесконечного множества.
Аноним 17/02/15 Втр 18:39:47 #217 №239886 
14241875872820.jpg
14241875872821.jpg
14241875872832.jpg
>>239781
>Если бы материя не имела пределов делимости (такое в принципе логически возможно, если бы не сука кварки и электроны), тогда бы, наверное, можно было говорить о континууме материи как об актуальной бесконечности материальных "точек". (Аналогично - с пространством-временем.)
>2015 год
>не знать о планковских величинах
>не знать о голографическом принципе
>не знать о предельной информационной емкости пространства
Планк, Хокинг и Бекенштейн смотрят на тебя как на кефирщика.
Аноним 17/02/15 Втр 19:50:13 #218 №239900 
>>239886
> планковские величины
> азаза, обезьянки не могут измерить, используя придуманную теорию, значит меньше не может быть
Так и говори что бесконечности бессмысленны, как и существование чего бы то ни было без относительного целеполагания. Но скорее всего область применения теории ещё не впору расширять, ибо ну оно нахуй и так живём норм.
Аноним 17/02/15 Втр 19:58:09 #219 №239903 
>>239886
>Планк, Хокинг и Бекенштейн смотрят на тебя как на кефирщика.
Ты тупой? Илы, можэт, у тэбя, дарагой, с рюсским йазыком праблэм, да? Там стоит сослагательное наклонение, блять! И специально для таких как ты, там отмечено про квантованность материи. Про планковские масштабы мне не заливай - я про них знаю достаточно, и не на уровне "ололо-скомбинировали из трех констант хернюльку десять-в-минус-тридцать-третьей", а на уровне чтения литературы по LQG.

Информационными емкостями и голографическим принципом тут тоже не маши, потому что для вменяемых людей этот принцип = малдасеновское AdS/CFT-соответствие ("голографию" только в качестве метафоры используют), которое является чисто теоретической схемой, полезной для некоторых расчетов и покамест буквальном смысле не подтвержденное на практике. Обчитаются, блять, популярных статеек про энтропию черных дыр и начинают, блять, себя жрущими печеньки Сасскиндами мнить.

И алсо, лучше по заданному вопросу про бесконечности давай чо-нить скажи, вместо того чтоб баянистые фотки постить.
Аноним 17/02/15 Втр 20:00:58 #220 №239904 
>>239903
Сасскинд ест печеньки?
Аноним 17/02/15 Втр 20:05:23 #221 №239906 
>>239903
>Ты тупой?
Зачем ругаешься? Объясни в чём не прав онон отстраннённо от себя и его персоны. Написал такой длинный по местным меркам пост, а ничего им не объяснил, кроме собственной охуенности. Тред думается не только тупых вопросов, но и вразумительных ответов, иначе зачем клаву давить. Если хочется показать ЧСВ, что ж пиздеть ты мастер.
мимокрок
Аноним 17/02/15 Втр 20:19:50 #222 №239913 
Искал о предельной информационной емкости пространства и первая ссылка оказалась на интересный космологический сайт и научпопную статью о "Информации в голографической Вселенной". Может кому будет интересно.
Аноним 17/02/15 Втр 20:20:39 #223 №239914 
>>239913
Забыл ссылку.
Информация в голографической Вселенной
http://www.modcos.com/articles.php?id=61
Аноним 17/02/15 Втр 20:22:06 #224 №239916 
>>>239906
>Зачем ругаешься?
>Объясни в чём не прав онон
Потому что не люблю, когда не вчитавшись в камент, начинают включать "кефирщиков" на пустом месте. Вот в этом он не прав.
По поводу остального - можно только догадываться, что он не одобрял идею актуальной бесконечности. Что ж, в этом я его поддерживаю.

>>239904
Ага, в какой-то своей видеолекции на ютубе жрал во время лекции.

>Если хочется показать ЧСВ,
Если бы я хотел его показать, то был бы трипфагом и в ответе про бесконечность начал бы пиздеть про голограммы, превозносить струны и кидаться прочими терминами, не имеющими отношения к проблематике вопроса (который больше философский, чем физический). Так что не надо.
Аноним 17/02/15 Втр 21:03:36 #225 №239931 
14241962162260.jpg
>>239781
>конструктивисты налагают на такие абстракции запрет
Ой, да не пизди. Если существует конечный конструкт (алгоритм) позволяющий строить такой объект, то конструктивисту норм. Ну типа:
1. Положить X(0) = 0, I = 0
2. Вычислить X(I+1) = X(I) + 1
3. Увеличить I.
4. Повторить 2.
То все ок, конструктивизм в наличии.
А вот "Положим что существует такое X(I), что для любого I>0: X(I+1) = X(I) + 1" констуктивисту уже не оче, ибо так легко можно ввести объект, способ построения которого не то что неизвестен, а очень даже не существует.
мимо-НЕ-конструктивист
Аноним 17/02/15 Втр 21:09:29 #226 №239933 
>>239931
>Если существует конечный конструкт (алгоритм) позволяющий строить такой объект
О том и речь вообще-то. Если существует алгоритм до сколь угодно большого шага, описывающий (в пределе) потенциальную бесконечность - то норм.
А актуальная бесконечность, "чистое существование" и т.п. - не норм.
Аноним 17/02/15 Втр 21:10:10 #227 №239935 
И, кстати, дефиниции объектов просто перечислением свойств, которыми они должны обладать вполне себе принимаются конструктивистами, если приводят к утверждениям о не существовании таких объектов (доказательства от обратного, если по-простому)
Аноним 17/02/15 Втр 22:48:17 #228 №239961 
14242024977720.jpg
14242024977731.jpg
14242024977742.jpg
>>239903
Извини, няша, не злись. Я специально немного набросил, добавив старых фоток мудрых отцов-основателей, чтоб немого подогреть ставшей несколько унылой дискуссию, и, как видно, небезрезультатно, судя по постам заинтересовавшихся.
>>239914
Да, хороший популяризаторский сайт, жаль только, что автор его давно уже не обновляет.
>>239778
>>239781
У Виленкина, в его книге «Мир многих миров», есть очень интересные размышления на тему бесконечности вселенной в контексте вечной инфляции.
Для тех, кто не читал, вкратце:
1. Есть огромная, экспоненциально раздувающаяся, но конечная, "внешняя" Вселенная, заполненная ложным вакуумом.
2. В ложном вакууме, из-за его нестабильности и квантовых флуктуаций, постоянно возникают области его распада, которые расширяются со сверхсветовой скоростью, и которые представляют собой "островные" вселенные, подобные нашей. Момент распада ложного вакуума для внутреннего наблюдателя из островной вселенной выглядит как Большой взрыв.
3. А вот теперь самая мякотка: хотя внешняя Вселенная имеет конечные размеры, и с точки зрения внешнего наблюдателя, также конечны островные вселенные, но со стороны внутреннего жителя пузырьковой вселенной ее размер бесконечен!
Прикол здесь в том, что инфляционное раздувание продолжается во времени неограниченно, а восприятие времени для внешних и внутренних наблюдателей отличается – для обитателя островной вселенной оно отсчитывается от Большого взрыва и возникновения его пузыря, который не является единомоментным для внешнего наблюдателя. Первый пик – картина извне, видно, что в каждый временной срез размер образовавшейся вселенной конечен. Второй пик – вид изнутри, и здесь размер своей вселенной в любой момент внутреннего времени для ее жителя актуально бесконечен, поскольку всё более отдаленные регионы, которые для него уже существуют (и из прошлого которых он будет принимать сигналы с течением времени), для внешнего наблюдателя только будут "достраиваться" во все более отдаленном будущем. Как если бы кто-то путешествовал в компьютерной игре по карте, которая неограниченно генерится по мере продвижения, и делал отсюда вывод, что игровой мир бесконечен, только для полноты картины надо понимать, что время на сервере и время в игре текут принципиально по-разному. Так что такой вот синтез потенциальной и актуальной бесконечности.
Аноним 17/02/15 Втр 23:43:22 #229 №239969 
>>239961
>Извини, няша, не злись. Я специально немного набросил, добавив старых фоток мудрых отцов-основателей, чтоб немого подогреть ставшей несколько унылой дискуссию, и, как видно, небезрезультатно, судя по постам заинтересовавшихся.

Да ок, я тож чот переборщил ну двачик же
Аноним 18/02/15 Срд 00:19:13 #230 №239975 
14242079532730.jpg
Обучаясь чему-то новому, например, арифметике или взятию интеграла, мы "программируем" свой мозг для выполнения определенных операций и получаем новый скилл.

Если обучение можно считать программированием, есть ли какие-то гипотетические способы очень быстрого запрограммирования? (Мы используем только органы чувств для "программирования", и то не все).

Может, разработки какие-то современные на стыке педагогики и нейробиологии?

Написал коряво, но смысл, думаю понятен.
Аноним 18/02/15 Срд 00:26:29 #231 №239977 
>>239975
Обучение медленное, потому что тебе нужно провести всю хуету, которую ты хочешь выучить, через критическое восприятие подсознания, которое думает "Ну ты что мне, опять покушать принёс?". Мотивацию ищи. Вся суть там.
Аноним 18/02/15 Срд 10:41:25 #232 №240010 
>>239975
ЛСД
Аноним 19/02/15 Чтв 10:59:09 #233 №240144 
Почему во время перехода электрона с менее выгодного на более выгодный уровень, он излучает фотон? Потому, что теперь для связывания протона и электрона требуется меньше сил, а фотон это как бы излишек?

Почему фотон движется в вакууме? Потому, что там есть электромагнитное поле, возбуждением которого является волна-фотон?

Вот, есть у меня, например, молекула АТФ. Я беру ее и, используя молекулу воды, отщепляю от хвоста одну молекулу фосфорной кислоты. В итоге у меня получается АДФ, ортофосфорная кислота и энергия. Какой вид имеет эта энергия и как ее можно использовать внутри клетки? Конформация атомов итоговых молекул такова, что им теперь требуется меньше сил на поддержание электростатических связей и избыток энергии в виде фотонов поглащается, например, какими-нибудь соседними электронами, которые от этого переходят на невыгодные высокие уровни, где попадают в радиус действия притяжения протонов и таким образом образуются новые связи/молекулы?
Аноним 19/02/15 Чтв 15:01:30 #234 №240176 
Науканы, поясните за жизнь по-хардкору.
Что в ней такого особенного, и почему мы должны ее искать где-то. Как по мне - так это хуевая затея. Что такое жизнь? Особый набор химических элементов в порядке, не более. Ничего особенного. Материя воссоздает саму себч. Почему это должно быть интересно нам?
Аноним 19/02/15 Чтв 15:18:59 #235 №240183 
>>240144
E2 - E1 = hv, разность энергий на различных уровнях.

Гугли волновое уравнение электромагнитного поля. Переменное электрическое поле вызывает переменное магнитное, которое в свою очередь вызывает электрическое... ну ты понел. Если не придётся совершать работу на пермещение заряда, например, то эти возмущения будут продолжаться бесконечно, ну и происходить со скоростью света.
Аноним 19/02/15 Чтв 15:31:03 #236 №240188 
14243490639720.jpg
>>240183
Аноним 19/02/15 Чтв 16:28:59 #237 №240200 
Надеюсь, тут есть органические химики.

Можно ли считать бромирование алкена за редокс-реакцию? Меняются же оксидативные числа атомов углерода и брома.

В тесте выбрал оксидацию, а в. Ответах указано, что это добавление на активированное двойное соединение. Олефин связан с двумя субституентами с -М-эффектом.
Аноним 19/02/15 Чтв 17:33:30 #238 №240213 
>>240144
> Почему во время перехода электрона с менее выгодного на более выгодный уровень, он излучает фотон? Потому, что теперь для связывания протона и электрона требуется меньше сил, а фотон это как бы излишек?

Энергетические уровни в атоме обладают разным значением энергии. Чтобы перейти с одного уровня на другой, электрону нужно испустить фотон - излишек энергии, или поглотить фотон - недостаток энергии. Разность энергии между двумя уровнями равна энергии фотона.

>Почему фотон движется в вакууме? Потому, что там есть электромагнитное поле, возбуждением которого является волна-фотон?

Фотон - частица, движется в вакууме потому, что такова его природа. Фотон движется со скоростью света так как обладает нулевой массой.

Аноним 19/02/15 Чтв 18:14:37 #239 №240220 
>>240213
Если фотон не имеет массы, как же тогда гравитационное поле черных дыр может искривлять его траекторию?
Аноним 19/02/15 Чтв 18:46:42 #240 №240225 
>>240220
Да имеет он массу. Это просто какая-то очередная условность про безмассовость чтобы новые теории натянуть на механику дидов.
Аноним 19/02/15 Чтв 18:50:12 #241 №240227 
>движется в вакууме потому, что такова его природа
Вообще охуеть.
Аноним 19/02/15 Чтв 20:19:12 #242 №240245 
>>240227
кефиромразь вскукарекнула
Аноним 19/02/15 Чтв 20:22:30 #243 №240247 
>>240245
Что ты несешь, наркоман?
Аноним 19/02/15 Чтв 20:25:33 #244 №240248 
>>240144
По первой части - все так, но после
>есть у меня, например, молекула АТФ.
Хуита пошла. Тут скорее вопрос к химикам/биологам, а я у мамки физик, но вот что я думаю. Фотоны тут не при чем. Внутренная энергия связей между частями молекулы переходит в их кинетическую энергию, а потом через столкновения с другими разогревает окружающую среду.
Аноним 19/02/15 Чтв 20:39:05 #245 №240253 
>>240248
>энергия связей между частями молекулы переходит в их кинетическую энергию
Кинетическую энергию чего и каким образом переходит?
>разогревает окружающую среду
Нахуя?
Аноним 19/02/15 Чтв 20:48:30 #246 №240254 
>>240253
> Кинетическую энергию чего
движения молекулы
> и каким образом переходит?
Заряды друг от друга отталкиваются
> Нахуя?
Даже не знаю что тебе сказать... чтоб не здохнуть, наверное.
Аноним 19/02/15 Чтв 20:52:04 #247 №240255 
14243683244330.gif
>>240254
Нет, прости, но как-то не клеится. Попробуй ознакомиться механизмом с движением моторного белка. Кинезина, например. Как раз на один его шаг тратится энергия одной молекулы АТФ. А пока твой ответ выглядит, скажем, не убедительно.
Аноним 19/02/15 Чтв 21:54:41 #248 №240265 
>>240255
Может быть, когда обе ноги касаются стебля, в заднюю ногу поступает молекула АТФ и происходит ее "подрыв". Тогда заднюю ногу отбрасывает вперед.
Чисто на правах гипотезы. Интересная тема, может еще почитаю.
Аноним 19/02/15 Чтв 22:02:44 #249 №240267 
>>240220
Потому что внезапно, ОТО утверждает, что нет никакого гравитационного поля, а есть искривление пространства-времени, порождаемое массивными телами. И в этом пространстве кратчайшими траекториями являются ни разу не прямые в привычном понимании этого слова
Аноним 19/02/15 Чтв 22:30:35 #250 №240274 
>>239251
Бамп
Аноним 19/02/15 Чтв 22:53:52 #251 №240282 
14243756326170.jpg
>>236711
Чисто теоритически. Если сесть в космический корабль, которые будет лететь в 100500 раз быстрее света и лететь тупо в одном направлении и преодолеть границы нашей Вселенной, куда мы попадём? В ту Вселенную, какой она была до Большого Взрыва? А она была вообще? И если там что-то будет, как появилось это всё, а не наша Вселенная конкретно?
Аноним 19/02/15 Чтв 23:33:51 #252 №240290 
Аноны, подкиньте книжек, статей, вводных учебников по взгляду на вселенную как на отображение из 4, 5 ... n мерного пространства в k-мерное, характеризующее состояние точки? Или это и есть теория струн вкраце?
Аноним 20/02/15 Птн 00:00:38 #253 №240295 
>>240282
Это невозможно чисто теоретически. Если ты будешь двигаться со скоростью, близкой к скорости света, то время на Земле будет медленнее, относительно твоего. Ты будешь как будто путешествовать в будущее.
Аноним 20/02/15 Птн 00:20:36 #254 №240300 
14243808367490.jpg
Доброй ночи, анон. Есть Одна Тупая задача, но я видимо ещё тупее.

Есть конвейер по которому движутся детали пронумерованные подряд от 1 до N, есть рабочий, у которого стоят коробки пронумерованные от 1 до M. M заведомо меньше N. Рабочий поочередно снимает детали и раскладывает их по коробкам. Например случай для 14 деталей и 3 коробок:
1 коробка: 1, 4, 7, 10, 13
2 коробка: 2, 5, 8, 11, 14
3 коробка: 3, 6, 9, 12
Требуется написать функцию от трех переменных (количество деталей, количество коробок, номер коробки), которая вычислит номер последней детали в коробке. Можно пользоваться арифметическими действиями и остатком от деления.

Я уже извел дохрена бумаги подбирая закономерность до шести коробок, но нихрена не вижу. Одни частные случаи.
Аноним 20/02/15 Птн 00:21:41 #255 №240301 
14243809011850.jpg
>>240265
Ну, в любом случае нужна энергия на то, чтобы ногу от тубулина "отклеить". Это очевидно же.
Но какова физика этого процесса?
Кроме того, что там за шарнир на котором вращается одна нога и за счет чего она вращается?
Тема очень интересная, так что непременно ознакомься и приходи сюда пояснять нормально с точки зрения физики.
Алсо, могу посоветовать в качестве начала учебник по молекулярной биологии Альбертса. Там кажется есть целая глава чисто по двигательным белкам. А уж про АТФ столько написано, что вообще пиздец.
Аноним 20/02/15 Птн 00:46:26 #256 №240305 
>>240300
Попробуй как-нибудь по индукции порассуждать.
И ещё помни о существовании функции sgn(x)
Аноним 20/02/15 Птн 00:52:34 #257 №240306 
>>240301
В статье на википедии
https://en.wikipedia.org/wiki/Kinesin
говорится что толком этот механизм не изучен
> Many challenges are encountered in theoretical investigations given the remaining uncertainties about the roles of protein structures, precise way energy from ATP is transformed into mechanical work, and the roles played by thermal fluctuations. This is a rather active area of research.
там же есть ссылка на довольно любопытную статью
https://www.ncbi.nlm.nih.gov/pmc/articles/PMC1281917
Так что пиздец как там по хитрому все устроено.
Но такие процессы происходят не за счет переизлучения фотонов инфа-99%
Аноним 20/02/15 Птн 00:56:12 #258 №240309 
14243829721020.jpg
>>240306
В вики хуйня написана. Вот просто прямо сейчас качай учебник Альбертса с рутора и открывай, кажется, 16ую главу и смотри насколько "не изучен" механизм работы двигательных белков, лол.

Почему не за счет? Что такого особого в излучении фотона? Просто электрон перейдя на более выгодный уровень отдал излишек энергии. Почему этого не может произойти в какой-то химической реакции, например, гидролизе АТФ?
Аноним 20/02/15 Птн 01:10:09 #259 №240311 
>>240306
>>240309
Глава 16.3.4, страница 1559
Аноним 20/02/15 Птн 02:22:29 #260 №240318 
>>240309
Посмотрел. Ни одной формулы. Изучили они, лол.

Излучить фотон можно, но не в этой реакции как ты его ловить будешь, это ведь совсем непросто?

Я так понял в результате реакции АТФ в ноге что-то перестраивается и она отлипает.
Не совсем понял как возникает цикличность.

Еще крайне забавный факт. Прочитал его в фейнмановских лекциях по вычислениям, что многие штуки типа таких двигаются не как в смешных картинках - только вперед, а перемещаются то туда - то сюда. Фишка в родственности понятий эффективности - обратимости, но это совсем отдельный вопрос.
Аноним 20/02/15 Птн 10:17:02 #261 №240354 
14244166220370.png
>>236711
why?
Аноним 20/02/15 Птн 12:00:03 #262 №240381 
>>240290
Бамп в тематике
Аноним 20/02/15 Птн 12:07:44 #263 №240384 
>>240318
>это ведь совсем непросто
Да, наверное, ты прав.

>Я так понял в результате реакции АТФ в ноге что-то перестраивается и она отлипает.
Не совсем понял как возникает цикличность.
Ну, ты разобрался, я смотрю.

>Ни одной формулы
Зачем там должны быть формулы? Они идут раньше, где объясняется, как это все работает на более простом уровне. И ты читая такой текст, по идее, самостоятельно должен представлять формулы, которых ему не хватает и которые стоят за описываемыми в тексте процессами.

>только вперед, а перемещаются то туда - то сюда
Ну да, так и есть. В клетке все процессы, конечно, подчинены общей идее и они происходят, но это выглядит совсем не как организованная работа каких-нибудь пролетариев на заводе.

В общем, я понял, если я и найду ответы на свои вопросы, то только самостоятельно.
Аноним 20/02/15 Птн 14:29:16 #264 №240430 
14244317569750.jpg
>>236711
http://lenta.ru/news/2015/02/20/bigsize/
>Ученые доказали направленность эволюции в сторону увеличения размеров животных
>Несмотря на столь значимые исключения, как гигантские динозавры, со времени возникновения жизни фауна планеты становится все больше по размеру.

я не понял, а как же насекомые и рептилии, мамонты и прочие ископаемые чудовища?
анон поясни, где я упустил
Аноним 20/02/15 Птн 15:07:43 #265 №240442 
>>240430
Нет, закон "все виды стремятся к увеличению" - лишь частный случай закона "все виды стараются выжить" для конкретных условий. Во многих экологических нишах преимущество даёт как раз уменьшение размеров: http://www.youtube.com/watch?v=NAXFHeM9BM0
Аноним 20/02/15 Птн 15:33:38 #266 №240447 
>>240442
т.е. британские учёные неправы?
Аноним 20/02/15 Птн 15:37:07 #267 №240448 
>>240430
Не ведись на хуйню. В стародавние времена гигантов на нашей планете было пиздец как много кислорода. А теперь мало, его количество уменьшается, и все становятся карланами.
Аноним 20/02/15 Птн 16:18:39 #268 №240457 
>>240448
>>240442
ладно спасибо котаны, по крайней мере я понял что я не один
Аноним 20/02/15 Птн 16:54:50 #269 №240461 
14244404907940.png
Допустим у нас есть: АК-74М , он же но для ОБЧР , размером с планету , звезду , галлактику.
Какой из них будет работать? Какой будет существовать?
2 вопроса! Аноним 20/02/15 Птн 18:10:26 #270 №240484 
14244450261040.jpg
1. Эволюционисты объясните ПОЧЕМУ люди видят т.н. "видимый свет" который бывает не всегда, а не видят инфракрасные лучи которые светят от всех предметов всегда и в них можно все всегда видеть не завися от источников света вращающихся вокруг Земли? Ведь люди полсуток считай слепые, и собаки например.

2. Коперники объясните зачем форсится идея что земля вращается вокруг солнца? Ведь все относительно и можно считать что солнце вращается вокруг земли, и даже вся вселенная.
Аноним 20/02/15 Птн 18:18:21 #271 №240491 
>>240484
>все относительно
Уйди.
Аноним 20/02/15 Птн 18:27:55 #272 №240497 
>>240484
> ПОЧЕМУ люди видят т.н. "видимый свет"
Потому что он дает больше инофрмации об объекте, чем ИК излучение.
> зачем форсится идея что земля вращается вокруг солнца?
Земля не вращается вокруг Солнца, Земля вращается вокруг центра масс. Помимо Земли вокруг него вращается еще до жопы тел, поэтому считать центром вселенной Землю не имеет смысла.
Аноним 20/02/15 Птн 18:28:33 #273 №240498 
14244461137970.png
>>240491
Кефирщик сам?
Аноним 20/02/15 Птн 18:35:45 #274 №240500 
>>240497
>Потому что он дает больше инофрмации об объекте, чем ИК излучение
Чем больше, у большинства существ даже цветного зрения нет, и света полсуток нет, существа слепые и беззащитные. Если он вообще есть, а например в зарослях его мало. В ИК все бы всегда все видели.
>вращается еще до жопы тел
То есть просто потому что так проще, это условность? Все равно же даже задача 3-х тел не решаема.
Аноним 20/02/15 Птн 18:41:17 #275 №240503 
>>240500
> Чем больше,
Всем больше, болезный. Даже монохромное зрение в видимом диапазоне позволяет оценить кучу параметров объекта, которые ты никак не оценишь по ИК спектру.
> То есть просто потому что так проще, это условность?
В каком-то смысле, да. о во вращении, скажем, лекого Марса вокруг тяжелого Солнца по эллиптической орбите есть физический смысл - понятно, какие силы куда направлены, и все такое. А вот в движении Марса вокруг Земли по ебанутой орбите физического смысла нет никакого.
Аноним 20/02/15 Птн 18:42:34 #276 №240504 
>>240500
>>240503
На самом деле намного интереснее, если бы люди как дельфины воспринимали отражение звука и строили на этом отражении образы.
Аноним 20/02/15 Птн 18:45:04 #277 №240508 
>>240504
То есть тебя не устраивает, что люди как люди воспринимают отражение света и строят на нём образы?
Аноним 20/02/15 Птн 18:46:06 #278 №240509 
>>240508
Это не очень полноценное восприятие. Мозг способен на большее.
Аноним 20/02/15 Птн 18:53:06 #279 №240512 
>>240509
Ты сначала то, что уже есть, осиль, унтерпримат.
Перестань выпадать из окон, попадать под машины, разбивать себе головы об унитазы, поскользнувшись на мыле. Из-за богатого внутреннего мира и широты взгядов.
Аноним 20/02/15 Птн 18:55:25 #280 №240513 
>>240512
Ни разу не делал ничего подобного. Но да, соглашусь, человечеству нужно избавляться от слепой зоны и раскачивать периферийное зрение.
Аноним 20/02/15 Птн 18:57:23 #281 №240514 
14244478431350.jpg
>>240503
>Даже монохромное зрение в видимом диапазоне позволяет оценить кучу параметров объекта, которые ты никак не оценишь по ИК спектру
Это какие? Тем более что может быть важнее видимости распределения тепла например?
Аноним 20/02/15 Птн 18:57:52 #282 №240515 
14244478721460.jpg
>>240513
>человечеству нужно избавляться от слепой зоны и раскачивать периферийное зрение.
Нет. Человечеству нужно избавляться от саморефлексии и раскачивать ловкость и скорость.
Аноним 20/02/15 Птн 18:59:15 #283 №240516 
>>240514
Зачем тебе видение тепла? Кусать кого-то за яремную вену хочешь?

>>240515
Я за механику. Впрочем, опыт игры в Spore показывает, что да, надо бы. Но генной инженерией хотя бы.
Аноним 20/02/15 Птн 19:00:28 #284 №240517 
14244480284210.jpg
Например еслиб люди видели в ИК сразу стало бы ясно по температуре (приливам крови) не пиздит ли исус.
Аноним 20/02/15 Птн 19:06:22 #285 №240519 
14244483829300.jpg
Здоровье лучше видно половых партнеров, реакции организма, даже думает ли чел или нет, мозг же печка. Лучшие теплые места выбирать, охотится и собирательство, а кстати растения тоже умеют менять температуру активно или пассивно, есть даже типа теплокровных. Мы же в мире тепловых процессов, а видим какой-то жалкий видимый спектр ни о чем почти не говорящий.
Аноним 20/02/15 Птн 19:09:25 #286 №240522 
>>240517
>>240519
А если бы у бабушки были яйца, она бы лучше понимала дедушку.
Без лжи человеческое общество распадётся быстрее, чем ты успеешь сказать "упс".
Аноним 20/02/15 Птн 19:20:37 #287 №240525 
>>240522
Но ведь эволюционисты считают что глаза зародились раньше лжи, а общество пережиток животных. И что у эволюции нет цели, и никакого если бы не учитывается.
Аноним 20/02/15 Птн 19:26:15 #288 №240528 
>>240525
А что, все на самом деле по-другому?
Аноним 20/02/15 Птн 19:43:21 #289 №240532 
>>240461
бамп
Аноним 20/02/15 Птн 19:51:03 #290 №240534 
>>240528
Нет, Иисус, я не поведусь хуедесятый раз.
Аноним 20/02/15 Птн 19:52:30 #291 №240535 
>>240532
Если он будет такой большой, то где взять такую большу руку что бы нажать на курок? Никакой работать не будет, никто не сможет воспользоватся им, только великан размером с галаткику.
Аноним 20/02/15 Птн 20:11:34 #292 №240537 
>>240535
Допустим он есть.
Аноним 20/02/15 Птн 21:18:43 #293 №240546 
>>240290
Братишки, ну ответьте же наконец!
Аноним 20/02/15 Птн 21:50:51 #294 №240560 
14244582518730.png
14244582518731.jpg
>>240354
отвечайте мой вопрос
и вот ещё один - можно ли название темы тоже скрывать? если я скрыл - наверное меня это вообще не интересует.
мод, поправь, пожалуйста (в d и abuy про pdf спрашивал - до сих пор не работает. пошустри пожалуйста по своим (может у абу сегодня пятничный шаббат, но завтра имеет смысл планёркой неделю начать)
пик для привлечения модератора
Аноним 20/02/15 Птн 21:52:45 #295 №240562 
>>240546
Вот тебе книга
Zwiebach - A first course in string theory
когда все прочтешь и во всем разберешься можешь задавать еще вопросы
Аноним 20/02/15 Птн 21:53:52 #296 №240564 
Если академик Павлов был атеистом, почему он завещал отпеть его по православному обряду после смерти?
Аноним 20/02/15 Птн 22:00:58 #297 №240567 
>>240564
Рефлекторно.
Аноним 20/02/15 Птн 23:00:31 #298 №240592 
>>240567
Зазмеился.
Аноним 20/02/15 Птн 23:54:52 #299 №240603 
>>240461
Бамп
Аноним 21/02/15 Суб 01:02:23 #300 №240613 
>>240484
Зрение появилось ещё когда мы жили в воде. Термографическое зрение отображало бы температуру воды. Да и теплокровных тогда не было, а когда глаз уже образовался, то он был покрыт непрозрачной для термографии роговицей.
Аноним 21/02/15 Суб 02:27:05 #301 №240621 
>>240384
> Зачем там должны быть формулы?
Я вот что подумал, там ведь и Броуновское движение вносит ощутимый вклад. Но чтобы понять насколько - тут как раз нужны числа, формулы, расчеты.
Это тебе не горох считать.
Аноним 21/02/15 Суб 09:47:01 #302 №240649 
>>240484
>ПОЧЕМУ люди видят т.н. "видимый свет"
Потому что это давало репродуктивные преимущества нашим предкам.
>не видят инфракрасные лучи
А это — не давало.
>идея что земля вращается вокруг солнца?
Вокруг общего центра масс, если точнее.
>зачем
Затем, что так и есть.
Аноним 21/02/15 Суб 09:51:22 #303 №240650 
>>240430
>читать ленту в 2015 году
>анон поясни, где я упустил
http://elementy.ru/
Аноним 21/02/15 Суб 10:24:15 #304 №240655 
>>240650
>2015
>читать elementy, когда есть arxiv, LibGen и сай-хаб
Аноним 21/02/15 Суб 10:51:04 #305 №240664 
14245050648290.jpg
>>240613
>Термографическое зрение отображало бы температуру воды
На ютубе есть инфрасъемки под водой вот например про крыбалку
http://www.youtube.com/watch?v=PE9FKX8Neaw
или вот что то про это
http://www.tshed.co.uk/1/news.php?item.45
алсо видимый свет под воду проникает не слишком хорошо, а в глубинах его вообще нет. Ну и опять же ночью его вообще нет, или если день не солнечный освещенность сильно падает, или из-за мути, в том числе если она где-то сверху.
>Зрение появилось ещё когда мы жили в воде
Все же потом много адаптаций появилось, жабры например сменились легкими, электрочувства почти пропали, ноги там и прочее отросло, специальные уши.
Аноним 21/02/15 Суб 10:54:26 #306 №240665 
>>240655
Это на чучмецком языке, да сайты то какие убогие в 15 году. Не выебывайся космополит.
Аноним 21/02/15 Суб 10:58:14 #307 №240666 
>>240649
>Вокруг общего центра масс, если точнее
Да ладно, помещаем в теории Землю в центр мироздания и все вокруг нее будет вращаться как миленькое. А тем более с точки зрения струн, бозонов и прочих потусторонних вещей это вообще один фиг эпициклы ничем не хуже коперничества.
Аноним 21/02/15 Суб 11:01:42 #308 №240667 
>>240649
>Потому что это давало репродуктивные преимущества нашим предкам.
>А это — не давало
Ага, а копченые все уши прожужжали про то что весь век динозавров млекопетающие были ночными зверьками, 150 миллионов лет пучили в темноту глаза.
Аноним 21/02/15 Суб 11:41:26 #309 №240679 
>>240484
>зачем форсится идея что земля вращается вокруг солнца? Ведь все относительно и можно считать что солнце вращается вокруг земли, и даже вся вселенная

Вопрос сводится к выбору системы отсчета. Гелиоцентрическая система отсчета удобнее для изучения динамики движения планет, потому что в ней траектории из птолемеевских (вырвиглазных и непредсказуемых) превращаются в красивые и легко рассчитываемые эллипсы, что выхватывает физическую суть, позволяет пейсать законы Кеплера и Ньютона в наиболее лаконичной форме, упрощает физическое описание, да и представляется наиболее наглядным и лаконичным.

Нет ничего невозможного и в том, чтобы выбрать геоцентрическую систему - да полно такого в той же наблюдательной астрономии, где есть "суточное вращение небесной сферы", звезды имеют "склонение и восхождение", Солнце "движется по линии эклиптики" и т.п. Для каких-то задач астрономии этот словарь оказывается более удобным и его можно встретить в любом учебнике астрономии.

Если же ты хочешь понять, что более физично - то это вопрос более хитрый. С точки зрения чистой кинематики принципиальной разницы между разными системами отсчета нет (вопрос только в удобстве описания траекторий). С точки зрения же динамики ньютоновской, по крайней мере принципиальная разница есть: привилегированными считаются инерциальные системы отсчета, где справедливы всякие там галилеевские принципы относительности, первые законы Ньютона и т.п. В реале абсолютно инерциальных нет, есть лишь разные степени приближения к ним. С этой точки зрения гелиоцентрическая система "более" инерциальна (хотя и тоже не до конца), чем геоцентрическая. Поэтому в ней-то все законы динамики планет записываются наиболее просто и удобно. С точки зрения же ОТО можно было бы сказать, что любые системы - инерциальные и нет - физически равноправны (это связано с принципом общей ковариантности ОТО в любых допустимых системах отсчета/координат). Другой вопрос в том, что не всякая система отсчета в принципе реализуема материальными линейками и часами в сколь угодно широкой области охвата. Напр., во вращающейся системе (геоцентрической) с какого-то расстояния скорость наблюдаемых тел точек на коорд. осях превысит с, поэтому ее уже нельзя реализовать абстрактными "жесткими" линейками. Тем не менее, для каких-то ограниченных областей она вполне применима.
Аноним 21/02/15 Суб 11:44:06 #310 №240680 
14245082461130.jpg
>>240679
Пик Линдафшица отклеился
Аноним 21/02/15 Суб 12:02:27 #311 №240685 
>>240679
Спасибо, теперь вопрос прояснен. Это имхо довольно важно.

спросивший анон
Аноним 21/02/15 Суб 13:19:22 #312 №240700 
14245139625530.jpg
>>240497
> Земля не вращается вокруг Солнца, Земля вращается вокруг центра масс.
Аноним 21/02/15 Суб 15:40:08 #313 №240725 
>>240700
Щито не так?
Аноним 21/02/15 Суб 17:56:18 #314 №240736 
Как разработчикам консолей удается сделать такое железо, чтобы оно в любом случае потянуло игры из будущего? Например, выходит игра в 2015 и идет на РС и PS3. А вот игра из 2017 не будет идти на РС, но пойдет на приставке. Как это? И почему не делают компьютеры по подобию приставки?
Аноним 21/02/15 Суб 18:22:32 #315 №240742 
>>240562
То есть это относится к теории струн?
Благодарю за наводку на книжку
Аноним 21/02/15 Суб 19:14:34 #316 №240759 
>>240736
Пилят мыльное кинцо в 2018 что бы его потянула консоль из 2014, а для пеки просто напросто не делают нормальной оптимизации чтобы ты купил новую нвидию
Аноним 22/02/15 Вск 16:07:17 #317 №241004 
Есть один старинный способ. Узнал его от физички еще когда в школе учился.
Берешь кастрюлю литров на 8-10, заполняешь ее водой так, чтоб 1,5-2 см оставалось до края, ставишь ее на шкаф и подсовываешь под нее книги или твердые журналы, бортик кастрюли должен быть плотно прижат к потолку. Подносишь к кастрюльке источник звука, больших наушников будет достаточно (надень их на кастрюлю как на голову). Самой музыки соседи не услышат, но эхо и басы будут долбить им уши до тех пор, пока не выключишь музыку.
Большой плюс данного метода в том, что тебе музыка никак не помешает, ибо даже очень тихое звучание для тебя, кастрюля приумножит и передаст верхним соседям.
Два года назад мой верхний сосед и его громко топающие дети искали источник шума 3 дня. Он даже спускался ко мне, вслушивался, звонил жене, которая ему подтверждала, что шум продолжается, и уходил ни с чем :)

Как это обьясняется, анон?
Аноним 22/02/15 Вск 21:33:03 #318 №241124 
Как понять, какая область физики тебе интересна, если научные книги пишутся весьма занудно?
Аноним 22/02/15 Вск 22:52:14 #319 №241140 
>>241124
Читай научно-популярные.
Алсо, если тебе учебники и научные книги кажутся занудными, тогда, вероятно, физика вообще не для тебя, потому что для занятий ею на профессиональном уровне тебе придется прогрызть тонны "занудной" инфы.
Аноним 22/02/15 Вск 23:10:46 #320 №241143 
>>241004
Это объясняется взаимным соседским мудачеством.
Аноним 22/02/15 Вск 23:12:13 #321 №241145 
>>240700
Фиха в том, что этот центр масс попадает внутрь объёма солнца.
Аноним 22/02/15 Вск 23:22:34 #322 №241147 
1) Что будет если нейтронную материю бомбить электронами?
2) Что находиться в белых карликах(или из чего состоит)? Написано, что еще не нейтронная материя, но пиздец как сжата. Если обычная плазма, которая очень сжата, то почему она не выгорает до железа-56 и дальше, там же под большим давлением и температурой термоядерная реакция наоборот должна ускоряться, а пишут что белый карлик будет жить дохуя лет, больше чем даже вселенная, даже Аллаха.
3) Что будет находиться на черных карликах(или из чего будет состоять) ?
4) Что именно находиться на поверхности нейтронных звезд? Та же хуита что и на белых карликах? Почему нейтронные звезды долго живут, ведь время жизни нейтрона составляет 15 минут? Но нейтронная звезда как и белый карлик будут жить дохуя лет, почему?
Аноним 22/02/15 Вск 23:24:19 #323 №241148 
>>241147
> ться
маи_глоза.жпг
Аноним 22/02/15 Вск 23:28:18 #324 №241151 
>>241148
А по по вопросу ответить можешь? Или ты ебаный гуманитарий который только и может что доебыватся до орфографических и пунктуационных ошибок?
Аноним 23/02/15 Пнд 00:01:23 #325 №241165 
>>241151
Ты же понимаешь, ебаный технарь, что и ебаный гумманитарий, и ебаный технарь - сорта говна, отбросов образования, не способных в науку? Такие, как ты, только и способны болеть за спартак/зенит/факультет.
Аноним 23/02/15 Пнд 00:54:07 #326 №241185 
14246420475050.jpg
>>240461
бамп
Аноним 23/02/15 Пнд 01:03:21 #327 №241191 
>>241147
1) ничего
2) выгорает@остывает
3) это ведь и есть остывший БК (вики)
4) наверное / только для свободного нейтрона, внутри ядер нейтроны тоже не распадаются
на права имхо
Аноним 23/02/15 Пнд 01:55:21 #328 №241228 
Мучает меня вопрос отношачек. Вот смотри анон, у меня не было и нет тянки, отношения с людьми всегда были отстраненными замкнутый по жизни. Значит ли это что части моего мозга, ответственные за социоблядство и эмпатию у меня слабо работают или вообще не работают? Правда ли что есть возраст когда себя уже не поменять?
Аноним 23/02/15 Пнд 02:23:34 #329 №241239 
>>241228
> Мучает меня вопрос отношачек.
Сажи биопроблемнику.
> Вот смотри анон, у меня не было и нет тянки, отношения с людьми всегда были отстраненными замкнутый по жизни.
Сажи инвалиду пунктуации.
> Значит ли это что части моего мозга, ответственные за социоблядство и эмпатию у меня слабо работают или вообще не работают?
Лучше называть это не частями, а функциями, т.к. они "размазаны" не только по отделам мозга, но и по биохимии. Да, в твоём мозгу эти функции ограничены, но не факт, что патологически. Т.е., ты можешь действительно оказаться аутистом, а можешь просто мальчуганом, неумело воспитанным обиженной на мужчин мамкой.
> Правда ли что есть возраст когда себя уже не поменять?
Смотря о каких изменениях ты говоришь. Вообще, на каком-то уровне ты меняешься всегда, ты никогда не в точности такой, какой был вчера. Каждая новая минута жизненного опыта пусть даже незначительно, но влияет на развитие твоего сознания. Но если говорить, например, о старых привычках, то да, над ними придётся поработать и потратить время, если решил их изменить. Принципиально возраста, отключающего функцию развития, нет, ты всегда способен меняться. Другое дело, что с возрастом ты всё больше и больше обрастаешь привычками, на изменение которых нужно тратить время, примерно равное времени жизни с этими привычками, и потому ты однажды можешь оказаться с "грузом прошлого", на переработку которого тебе потребуется времени больше, чем тебе осталось жить. Хотя и борьба с этим грузом уже само по себе - изменение.
Аноним 23/02/15 Пнд 03:05:51 #330 №241254 
14246499511830.jpg
>>236711
кто там спрашивал, что такое спин?

http://www.popmech.ru/science/11510-shakhmaty-prostranstva-treugolnik-na-treugolnike/#full

не состояние частицы lol а скорее состоянии ландшафта микромира
Аноним 23/02/15 Пнд 03:24:26 #331 №241260 
Анчоус, у меня вопрос по космической теме.
Как (на чем и каким образом) американцы приземлялись на Луну и улетали с нее? Все с помощью одного и того же аппарата?
Аноним 23/02/15 Пнд 03:31:25 #332 №241261 
>>241239
>Принципиально возраста, отключающего функцию развития, нет, ты всегда способен меняться.
Но почему тогда невозможно перевоспитать детей маугли и влить их в общество?
другой анон
Аноним 23/02/15 Пнд 04:11:26 #333 №241264 
>>241261
Потому что научение речи (и другим базовым социальным навыком) происходит в большей степени путём импринтинга, а не сознательным усилием, и этот механизм действительно имеет очень ограниченный срок жизни (7 - 14 лет). Этот механизм основан не на запоминании, а на установлении нужной топологии связей в ещё растущих нейронах в молодом мозгу.
Аноним 23/02/15 Пнд 04:20:39 #334 №241265 
>>241264
7-14 лет - это предельный возраст переобучаемости маугли, но бОльшая часть связей в мозгу уже сформированы годам к трём (и продолжается изменяться до семи лет уже незначительно).
Аноним 23/02/15 Пнд 04:42:39 #335 №241268 
>>241260
вывели на орбиту земли одной йоба ракетой корабль аполон и лунный модуль, разеденили, повернулись на 180, сцепились носами, газанули подняв апогей орбиты к луне, возле луны еще раз газанули чтобы выйти на круговую орбиту, перелезли в лунный модуль, отсоеденились, тормознули, сели на луну, флаг, камни, фоточки, взлетели, оставив на поверхности ноги и посадочные двигатели и баки, состыковались с аполоном, улетели на земляшку

както так если коротко
Аноним 23/02/15 Пнд 08:57:42 #336 №241285 
14246710624040.jpg
Доброанон, объясни, пожалуйста, как вот у этих физиков получилось "сфотографировать" электронные орбитали?
http://www.strf.ru/material.aspx?CatalogId=21731&d_no=74190
Я думал, что такое невозможно сделать из-за неопределенности Гейзенберга. И сейчас так же думаю.
Аноним 23/02/15 Пнд 09:02:43 #337 №241286 
>>241285
>неопределенности Гейзенберга
Значение знаешь?
Аноним 23/02/15 Пнд 09:36:00 #338 №241288 
14246733603500.png
>>241286
Устанавливает предел точности одновременного определения пары характеризующих систему квантовых наблюдаемых, в данном случае координаты и импульса электрона. Может я ошибаюсь и данная погрешность не существенна в таких масштабах?
Аноним 23/02/15 Пнд 12:16:08 #339 №241323 
>>241285
В твоей же статье

"Электронная структура атома вольфрама на острие зонда. Изменение расстояния между атомом вольфрама на острие зонда и атомом углерода поверхности графита позволяет «прощупывать» различные орбитали электронов в атоме вольфрама. Изображения получены Александром Чайкой в ИФТТ РАН на микроскопе GPI-300. Указаны масштабы по горизонтали и вертикали – 30 пикометра (0,03 нм)"
Аноним 23/02/15 Пнд 12:41:33 #340 №241330 
>>236711
Если космос такой охуенный термос, то почему на ночной стороне планет и спутников без атмосферы тах холодно? Неужели через излучение так быстро проебуется все тепло?
Аноним 23/02/15 Пнд 12:44:15 #341 №241332 
>>241330
>Неужели через излучение так быстро проебуется все тепло?
Да.
Аноним 23/02/15 Пнд 13:47:26 #342 №241348 
>>236711
Температура вакуума около 2.7 градусов Кельвина?
Аноним 23/02/15 Пнд 13:49:58 #343 №241351 
>>241348
Нет, это температура реликта.
Аноним 23/02/15 Пнд 14:10:29 #344 №241362 
>>241351
Но он же в вакууме
Аноним 23/02/15 Пнд 14:15:21 #345 №241367 
А вот ученые массачусетского технологического институтяа попробовали охладить глубокий вакуум до минус миллион градусов. Для этого они зделоли специальный хохлодильник, где вмест жидкого хреона залит жидкий гелий при минус 274 кельвина. Дело простое - раз жидкий гелий и так ахуительно холодный, то хохлодильник охладит вакуум еще встократ. В нашем случае задача была получить минус миллион градусов, чтобы охлаждать реактор холодного термоядерного синтеза с температурой реакции в мгнитной колбе плюс двадцать миллионов градусов.
Так значит загудели турбеины, крыльчатка из каленого бериллия прокачивает жидкий гелий в урановые трубки, которые выдерживают температуру в минус миллион градусов, не превращаясь плутоний. Очень хороший материал этот катаный ураниемум. Через месяц раскрутки 10000 насосных турбин температура в обменнике достигла минус 999999 градус, остался последний градус до миллиона. И тут оператор хохлодильной установки заметил неладное. Тревожная вибрация сотрясла золотые коронки зубов в рту оператора синхрофазатронной хохлодильной установки получения сверххолодного ваккумма. Тьма начала сгущчаться, назревало зловещщее...


продолжение следует
Аноним 23/02/15 Пнд 14:54:27 #346 №241378 
>>241367
>минус 274 кельвина
Лол, дальше ни читал
Аноним 23/02/15 Пнд 15:38:46 #347 №241385 
>>241378
Можно дальше не читать уже после этой фразы
>охладить глубокий вакуум до минус миллион градусов
Аноним 23/02/15 Пнд 15:49:58 #348 №241388 
>>241348
Ученые различают несколько разных вакуумов. Технический вакуум, физический вакуум, ложный вакуум, а еще есть эйнштейновский вакуум. Тебя какой интересует?
Аноним 23/02/15 Пнд 15:51:32 #349 №241389 
>>241330
Если нет атмосферы, или она крайне разрежена.
Аноним 23/02/15 Пнд 16:22:47 #350 №241394 
Электрон "не падает" в ядро потому, что для этого ему надо было бы обладать несравнимо большим импульсом. И хотя электрону можно сообщить какое-то количество энергии, прежде, чем ее хватит для уничтожения атома, он преодолеет электростатическое притяжение и покинет атом.
Так?
Аноним 23/02/15 Пнд 17:28:33 #351 №241415 
>>241285
>"не падает" в ядро потому, что для этого ему надо было бы обладать несравнимо большим импульсом

а почему тогда электрон, удаляется от ядра, если поглотит фотон и приближается, если испустит? наоборот же должно быть обьясните, почему я хуй
Аноним 23/02/15 Пнд 17:30:37 #352 №241416 
>>241394
>Так?
Нет.
Аноним 23/02/15 Пнд 17:31:03 #353 №241417 
>>241416
А как?
Аноним 23/02/15 Пнд 17:45:00 #354 №241420 
>>241417
>несравнимо большим импульсом
Наоборот. Рулетку видел когда-нибудь?
А не падает электрон на ядро потому, что принцип неопределенности не пускает. Гугли Бора, Гейзенберга, вот это все.
Аноним 23/02/15 Пнд 17:48:45 #355 №241421 
>>241420
>принцип неопределенности не пускает.
Пиздец, а я о чем по-твоему? Для локализации на таком маленьком участке нужен очень большой импульс и это как раз следует из принципа неопределенности. Протон же тяжелее электрона в овер 1.5к раз. Поэтому и импульс у него больше(масс на скорость же, епт). За счет этого он такой "маленький", т.е. границы его облака вероятностей маленькие. Соответственно, чтобы электрону оказаться в таких же малых границах, нужен импульс побольше. Ну и прежде, чем импульс станет большим, энергии будет достаточно для выхода за пределы атома. При чем тут рулетка?
Аноним 23/02/15 Пнд 17:50:47 #356 №241422 
>>241415
По-моему, он нихуя не удаляется от у него в прямом смысле. Он переходит на другой энергетический уровень - это да. И это обычно на модели с шариками изображают, как переход на более высокую орбиту. Но электрон же не шарик и орбит там никаких нет.
Аноним 23/02/15 Пнд 19:01:54 #357 №241434 
Ананас, а вот поясни мне такую штуку. Квантовая механика строится на двух принципах. Принцип неопределенности Гейзенберга и принцип квантовой запутанности. Как интерпретировать, т.е. объяснить эти 2 принципа?
С принципом неопределенности Гейзенберга вроде более менее понятно. С точки зрения многомировой интерпретации он отметается. В момент измерения (воздействия на частицу измерительным прибором) вселенная разделяется, реализует все возможные состояния частицы и в каждой вселенной свой измерительный прибор фиксирует конкретное положение этой частицы. Все логично и понятно.
Но как интерпретировать квантовую запутанность? Как взаимодействуют две квантово-запутанные частицы? Стандартная модель говорит нам, что все взаимодействия происходят посредством частиц-переносчиков этих взаимодействий и ограничены скоростью света. Но у запутанных частиц взаимодействие происходит мгновенно! Там не может быть никаких частиц переносчиков. Это просто рвет мне шаблон и не дает уснуть. Просвяти меня ананас!
Аноним 23/02/15 Пнд 20:45:21 #358 №241468 
14247135217360.jpg
>>241434
> вселенная разделяется
> Все логично и понятно
Это такой тонкий вброс?
Но если тебя удовлетворяет такое мировоззрение, то можешь думать о квантовой запутанности как о ниточке в гиперпространстве между двумя фотонами. На самом деле никто не знает как работает квантовая запутанность.
Аноним 23/02/15 Пнд 20:50:11 #359 №241470 
>>241434
Многомировая интерпретация, увы, не решает всех проблем интуитивного постижения квантового мира. Измерение и донесение информации до наблюдателя всё так же остаётся неформализованным процессом. Как и сам наблюдатель.

В квантовой запутанности нет переносчиков взаимодействия, две запутанные частицы становятся единым квантовым объектом. Эти частицы не взаимодействуют друг с другом, это одна частица с двумя координатами. И принцип неопределённости Гейзенберга (невозможность пронаблюдать свойство частицы без случайного изменения этого свойства) делают невозможным передачу информации со сверхсветовой скоростью с помощью такой растянутой в пространстве частицы.
Аноним 23/02/15 Пнд 21:28:44 #360 №241485 
>>241470
>Измерение и донесение информации до наблюдателя всё так же остаётся неформализованным процессом. Как и сам наблюдатель.
Ненене, погоди. Это в копенгагенской интерпретации наблюдателю отведена ключевая роль. В ММИ наблюдатель никакой роли не играет. Там все завязано на процессе измерения. И вот этот процесс измерения четко формализован. Суть его в том, что измерительный прибор воздействует на объект измерения (частицу). И вот это взаимодействие детектора (измерительного прибора) и частицы и является причиной разделения миров.
>В квантовой запутанности нет переносчиков взаимодействия, две запутанные частицы становятся единым квантовым объектом. Эти частицы не взаимодействуют друг с другом, это одна частица с двумя координатами. И принцип неопределённости Гейзенберга (невозможность пронаблюдать свойство частицы без случайного изменения этого свойства) делают невозможным передачу информации со сверхсветовой скоростью с помощью такой растянутой в пространстве частицы.
Нарушается принцип локальности. Есть ли какая-нибудь интерпретация вот этой растянутой частице с двумя координатами? Вот википидоры в этой статье https://ru.wikipedia.org/wiki/Квантовая_запутанность пишут:
>Многомировая интерпретация позволяет[56] представить запутанные частицы как проекции всех возможных состояний одной и той же частицы из параллельных вселенных.
В англ википедии написано, что в ММИ принцип локальности не нарушается. Но у меня с англ. не особо хорошо, может кто-нибудь донести до меня суть того что написано здесь?
http://arxiv.org/abs/quant-ph/0103079
Аноним 23/02/15 Пнд 21:39:03 #361 №241487 
>Многомировая интерпретация позволяет[56] представить запутанные частицы как проекции всех возможных состояний одной и той же частицы из параллельных вселенных.
Как это блядь понять???7711 Я себе уже весь могз сломал
Аноним 23/02/15 Пнд 21:41:56 #362 №241489 
>>241485
> Ненене, погоди. Это в копенгагенской интерпретации наблюдателю отведена ключевая роль. В ММИ наблюдатель никакой роли не играет. Там все завязано на процессе измерения.
Точно так же, как в копенгагенской.
> И вот этот процесс измерения четко формализован.
Нет, точно так же неформализован.
> Суть его в том, что измерительный прибор воздействует на объект измерения (частицу). И вот это взаимодействие детектора (измерительного прибора) и частицы и является причиной разделения миров.
Прибор тоже состоит из квантовых частиц. Кота Шрёдингера изучай.
> Нарушается принцип локальности.
Нарушается. Но вряд ли это большая катастрофа для тебя.
> Есть ли какая-нибудь интерпретация вот этой растянутой частице с двумя координатами?
Да, струна.
> Вот википидоры в этой статье https://ru.wikipedia.org/wiki/Квантовая_запутанность пишут:
> Многомировая интерпретация позволяет[56] представить запутанные частицы как проекции всех возможных состояний одной и той же частицы из параллельных вселенных.
Эта интерпретация эквивалентна струнной, она ничего не добавляет и не устраняет в этом вопросе. Для тебя проекция множества вселенных в точку является более интуитивной, чем растянутая в пространстве струна? Ок, пользуйся такой интерпретацией, это вопрос вкуса.
> В англ википедии написано, что в ММИ принцип локальности не нарушается.
Принцип локальности для тебя интуитивен? Почему ты боишься его нарушить?
> Но у меня с англ. не особо хорошо, может кто-нибудь донести до меня суть того что написано здесь?
> http://arxiv.org/abs/quant-ph/0103079
Там написано нечто, недоступное с уровня нашей дискуссии.
Аноним 23/02/15 Пнд 21:53:16 #363 №241495 
Вообщем ладно, я понял. Это не тема треда "тупых вопросов". Это тема рубрики "задай вопрос кандидату физико-математических наук".
Аноним 23/02/15 Пнд 21:59:03 #364 №241498 
>>241495
И потрать перед этим хотя бы пару лет на изучение базовых учебников.
репостну из бе Аноним 24/02/15 Втр 02:11:37 #365 №241561 
14247330974010.png
Анон, вот движение/распространение инфы выше скорости света невозможно, хорошо.
Возьмём очень длинный ряд лампочек с малым(не столь важно, важнее чтобы оно у всех ламп было одинаковым) временем включения.
То есть, за доли секунды лампочка загорается.

К каждой лампочке подключим систему включения с синхронизированными между всеми лампами часами.

Допустим, ширина одной лампочки с учётом зазора ~1см, на метре помещается сотня, на расстоянии, проходимом светом за секунду 29 979 200 000 лампочки.

А теперь, синхронизируем систему включения так, чтобы каждая последующая лампочка включалась на 1/29979200000 секунды позднее предыдущей.

По идее, мы получим, что весь ламповый путь у нас за секунду включается "со скоростью света".

Анон, а какую картину увидит внешний наблюдатель, если это время запаздывания включения последующей лампочки уменьшить, скажем, сделать в полтора раза короче.

Тогда мы увидим шлейф, распространяющийся быстрее скорости света?

Ну да, есть проблемы с синхронизацией и позицией наблюдателя (предположим, что он сидит в центре окружности, по которой расположены лампы так, чтобы свет от них до наблюдателя проходил одинаковое расстояние).
Аноним 24/02/15 Втр 02:23:03 #366 №241563 
>>241561
Да. Солнечный зайчик тоже так может.
sageАноним 24/02/15 Втр 02:51:12 #367 №241570 
>>241563
http://www.youtube.com/watch?v=JodrGzqHnN4
sageАноним 24/02/15 Втр 03:01:26 #368 №241573 
>>241561
лол что за система синхронизирования, максимальная скорость взаимодейсвтия-скорость света, лампочка просто не сможет загореться менее 1/29979200000 секунды позднее предыдущей потому что не получит на это сигнал
Аноним 24/02/15 Втр 05:16:54 #369 №241578 
14247442142420.jpg
>>241561
>движение/распространение инфы выше скорости света невозможно
КАК ЖЕ ВЫ ЗАЕБАЛИ ФОРСИТЬ ЭТО СУЕВЕРИЕ
никаких пруфов на это нет
Аноним 24/02/15 Втр 06:09:41 #370 №241579 
>>241578
>этот кефирный школотун
Ясно.
Аноним 24/02/15 Втр 06:22:40 #371 №241580 
>>241579
>БЕСПРУФНАЯ ВЕРА РЕЛЯТИВИСТОБЛЯДКА
ну побейся еще лбом
Аноним 24/02/15 Втр 06:24:04 #372 №241581 
>>241580
>этот мелкобуквенный БОЗОН
Понятно.
Аноним 24/02/15 Втр 09:33:45 #373 №241599 
>>238581
>тем же успехом можно было заявить, что мы не можем утверждать о несуществовании логически противоречивых понятий типа круглого квадрата

Я, может, слишком поверхностно знаком с логикой, но я что-то не пойму, где логическое противоречие в понятии "круглый квадрат".
Аноним 24/02/15 Втр 09:36:13 #374 №241600 
>>238674
>1) Что такое душа?
Религиозно-мифологическое понятие, значение которого разнится от системы к системе.
Аноним 24/02/15 Втр 10:05:35 #375 №241603 
14247615353820.jpg
Наверное платина, но когда произошёл большой взрыв, на его месте должна была остуться охуенная чёрная дыра. Самая охуенная на свете. Её ищет кто-нибудь вообще?
Аноним 24/02/15 Втр 10:18:56 #376 №241605 
>>241603
Вселенная находится внутри неё. Только нужно понимать условность этой фразы - чисто математически наша вселенная эквивалентна чёрной дыре, но вне этой дыры ничего нет. Эквивалентность заключается в принципиальной невозможности изнутри покинуть её.
Аноним 24/02/15 Втр 10:45:01 #377 №241608 
>>241603
Ничтожная вероятность того, что эта поебень находится в наблюдаемой нами Вселенной может служить одним из вариантов ответа?
Аноним 24/02/15 Втр 10:49:40 #378 №241610 
>>241608

А куда она денется? Болтается внутри какой-нибудь огромной галактики наверное.
Аноним 24/02/15 Втр 10:53:03 #379 №241611 
>>241610
ну это я к инфляционной модели подвожу. Вселенная же на многие порядки больше чем то, что мы можем видеть в радиусе 13.75 млрд. св. лет?
Аноним 24/02/15 Втр 10:56:04 #380 №241612 
>>241611
>Вселенная же на многие порядки больше чем то, что мы можем видеть в радиусе 13.75 млрд. св. лет?
Да.
Аноним 24/02/15 Втр 10:58:08 #381 №241613 
>>241612
а, ну значит я ещё не настолько отупел, всё правильно понял с:
Аноним 24/02/15 Втр 11:04:48 #382 №241614 
>>241603
>когда произошёл большой взрыв, на его месте должна была остуться охуенная чёрная дыра
Откуда такой вывод? Какая теория или гипотеза об этом говорит?
Аноним 24/02/15 Втр 11:53:18 #383 №241618 
А почему вообще возникают вопросы насчет того "как частица узнает, что её наблюдает наблюдатель - мистика ээкстрасенсорика бесплатно без сам" - ведь чтобы пронаблюдать частицу на неё надо повоздействовать полем там или ещё чем. Даже в макро-мире надо посветить на стенку, чтобы увидеть её - т.е. какое-то мини воздействие всегда есть.
Аноним 24/02/15 Втр 12:19:46 #384 №241621 
>>241614

Ну если после взрыва звёзд образуются эти дыры, то после взрыва объекта большей массы должна образоваться бОшльшая дыра? Или когда это всё взрывалось, то массы ещё не было?
Аноним 24/02/15 Втр 12:44:48 #385 №241622 
>>241621
Процесс коллапса звезд в ЧД, после взрыва сверхновых описывается общей теорией относительности. Но ОТО не может описать то, что было в момент Большого взрыва, а также в планковскую эпоху. ОТО не полна и для таких экстремальных условий неприменима.
Аноним 24/02/15 Втр 12:56:16 #386 №241625 
>>241599
Нарисуй.
Аноним 24/02/15 Втр 13:10:18 #387 №241627 
Здравствуй, лучший анонимный ученый уровня /sci. У меня непонятка. В основе разложения вектора на проекции лежит постулат о возможности представить вектор суммой нескольких других векторов. Так почему же проекции - скаляры?
Аноним 24/02/15 Втр 13:19:22 #388 №241628 
>>241627
Нет, они скаляры, домноженные на единичные вектора или сами собой вектора.
Аноним 24/02/15 Втр 13:37:17 #389 №241629 
>>241628
Проекция вектора на ось это тоже вектор. Вектор нельзя выразить одним числом. Следовательно скаляром эта проекция не является.
Аноним 24/02/15 Втр 14:38:36 #390 №241636 
Почему до сих пор нет прибора, который бы использовал ультразвук и позволял бы слепым людям видеть подобно летучим мышам? В чем загвоздка?
Аноним 24/02/15 Втр 15:18:13 #391 №241641 
Большой взрыв - это не реальное событие, это просто математическая абстракция. Вселенная описывается современной физикой как расширяющаяся, т.к. по-другому в математике пока нет возможности выразить динамические параметры пространства и материи. Потому такие формулы приводят к такому артефакту как Большой Взрыв, но это просто "предел видимости" описания физических законов, не более того.
Аноним 24/02/15 Втр 15:20:51 #392 №241644 
>>241636
Проблема не в создании прибора, а в том как этот прибор подключить к мозгу человека, имплантировать.
Аноним 24/02/15 Втр 15:46:22 #393 №241650 
Привет, наукач. Поясни мне вот что. Со школы помню, что чтобы определить, делится ли число на 3, нужно сложить все его цифры, и если результат на 3 делится, то и исходное число тоже. Так вот, как работает эта ебаная нумерология? Есть закон или просто так получилось)?
Аноним 24/02/15 Втр 16:02:52 #394 №241651 
Сап, ученые!
Хочу купить дипропиленгликоль (используется в косметологии, а также в печатном деле), нужен для того, чтобы изготовить ароматические палочки лол все эти порошки при пропитке им затвердевают на основе, и выходят эти самые долбаные палочки.
В местной нищесранской конторе, занимающейся поставками химреактивов, предложили просто пропиленгликоль. В аптеке сказали, что узнают о возможности заказать. Подскажите, может быть в какой-нибудь еще отрасли он активно используется? Просто бы пришел и купил у тех, кто его юзает, или вышел бы на поставщиков. inb4: в гугле забанили, пидор?
Аноним 24/02/15 Втр 16:13:11 #395 №241652 
>>241388
который в космосе
Аноним 24/02/15 Втр 16:15:19 #396 №241653 
>>241650
https://ru.m.wikipedia.org/wiki/%D0%9F%D1%80%D0%B8%D0%B7%D0%BD%D0%B0%D0%BA_%D0%9F%D0%B0%D1%81%D0%BA%D0%B0%D0%BB%D1%8F
Аноним 24/02/15 Втр 16:42:02 #397 №241654 
Что эти проценты в адресе википедии обозначают?
Аноним 24/02/15 Втр 17:10:46 #398 №241657 
>>241654
Unicode
Аноним 24/02/15 Втр 17:40:46 #399 №241663 
14247888460230.jpg
Сап, наукачи.
Объясните в метод Кранка-Николсона для решения нестационарного одномерного уравнения Шрёдингера.
Что расщеплять в волновой функции и гамильтониане? Координату? Тогда при перемножении матрицы гамильтониана и вектора волновой функции получится всрата слау, а не трёхдиагональная, которую должно решать прогонкой. Диагонализация матрицы расщеплённого гамильтониана? Но мы же расщепляем её методом конечных разностей, а не строим полноценную матрицу.
И что делать с мнимой единицей?
Аноним 24/02/15 Втр 18:44:29 #400 №241676 
>>241573
Они не подключены к общей шине питания/управления, а независимо запроганы на нужное время.
Аноним 24/02/15 Втр 19:06:06 #401 №241682 
>>241487
Хули непонятно то, блядь?
дурачок с вопросом про деление на тройку
Аноним 24/02/15 Втр 20:25:16 #402 №241697 
>>241663
Волновая функция должен быть вектор со значеними в дискретных точках по х. Гамильтониан включает вторую производную по х, поэтому приближение будет трехдиагональная матрица в натуре - будет брать значения в х(k+1), x(k-1) , x(k) чтобы получить вторую производную, плюс потенциал в точке x(k).
Аноним 24/02/15 Втр 20:50:59 #403 №241704 
>>241185
бамп
Аноним 25/02/15 Срд 16:46:51 #404 №241836 
>>241487
>Как это блядь понять???7711 Я себе уже весь могз сломал
Очевидно жи. Многомировая интерпретация позволяет представить твою мамку как проекции всех возможных состояний одной и той же твоей мамки из параллельных вселенных. То есть, в одной вселенной она совсем шлюха, в другой наоборот. А в целом (наша вселенная) получается нечто среднее.
Аноним 25/02/15 Срд 17:45:35 #405 №241856 
>>241836
Суперпозиция раком мамки?
Аноним 25/02/15 Срд 21:25:32 #406 №241887 
>>241641
Двачую этого будду.
Аноним 25/02/15 Срд 21:28:21 #407 №241889 
>>241618
Я тоже считаю, что йоба всё охуенно тогда пояснил с мячиками и клочками тумана. Не смог нагуглить, у тебя есть ссылка на тот тред?
Аноним 25/02/15 Срд 23:00:25 #408 №241898 
14248944251970.jpg
Что такое темная материя и темная энергия?
Аноним 25/02/15 Срд 23:04:33 #409 №241899 
>>241898
Никто с уверенностью не знает.
С неуверенностью - гоу ту вики, там тебе объяснят всяко лучше, чем пересказы тамошнего здесь же.
Аноним 26/02/15 Чтв 01:34:03 #410 №241922 
14249036439930.jpg
Платина.
1. Черные блядь дыры. Раз они есть - значит все пространство это ебаный мешок, за пределами которого что-то теоретически есть. Так?
2. Человечество способно создать такой корабль, который не расщепился бы нахуй, при вхождении в нее?
3. Где находится ближайшая к Земле черная дыра и сколько до неё добираться?
Аноним 26/02/15 Чтв 06:43:31 #411 №241927 
14249222115500.jpg
Не нашел геологии-треда, поэтому напишу здесь.

Аноны-геологи, посоветуйте каких-нибудь годных учебников или пособий по предмету, так же годных сайтов, научных фильмов на тему по сей чудесной науке. Я в ней полный ноль, но щас почитав немного про ледниковые периоды, климат и пустыни, понял что загорелся.

социолог-кун
Аноним 26/02/15 Чтв 07:51:29 #412 №241929 
>>241922
1. Нет, не так. Не понятно, почему ты такой вывод сделал?
2. Нет, не способно. Человечество вообще не знает, что находится за пределами горизонта событий. Общепризнаной, непротиворечивой теории на этот счет нет.
3. Центр нашей галактики. Сверхмассивная ЧД, ~70 млн. св. лет.
Аноним 26/02/15 Чтв 08:28:10 #413 №241933 
14249284905280.jpg
Какой сплав обладает самой большой ударной вязкостью?
Аноним 26/02/15 Чтв 08:29:18 #414 №241934 
>>241927
>почитав немного про ледниковые периоды, климат и пустыни, понял что загорелся
А мне больше нравятся камни, породы и то как они возникли. Вот например пещера, в ней на стене висит сосулька из чего-то, так мне очень интересно знать из чего и почему она выросла и как росла и вообще всю физико-химию ее понимать.
Аноним 26/02/15 Чтв 08:39:20 #415 №241935 
>>241933
Наверное, какой-нибудь сорт танковой брони.
Аноним 26/02/15 Чтв 08:54:47 #416 №241936 
>>241935
А конкретнее интересует точный состав. К тому же наверняка существуют сплавы на основе редкоземельных металов у которых УВ будет выше, чем у танковой брони.
Аноним 26/02/15 Чтв 08:57:10 #417 №241937 
>>237371

А если его толкнуть в пространстве пустом? Он же будет вечно ускоряться пока не достигнет скорости света, нет?
[http://www.youtube.com/watch?v=0n3OepDn5GU] Аноним 26/02/15 Чтв 09:08:45 #418 №241938 
>>236711
Таки Девид Дойч прав со своей теорией мульиверса( ну что типа есть паралельные вселенные, которых бесконечное множество)?
Аноним 26/02/15 Чтв 09:13:02 #419 №241939 
>>241937
lim->Inf =/= Inf
Аноним 26/02/15 Чтв 09:13:58 #420 №241940 
>>241938
Нет, если ты пытаешься обосновать его квантовой физикой.
[http://www.youtube.com/watch?v=0n3OepDn5GU] Аноним 26/02/15 Чтв 09:27:55 #421 №241947 
>>241940
Так общей теории всего(обоснованной) пока не существует?
Аноним 26/02/15 Чтв 09:41:28 #422 №241949 
>>241937
Лол, откуда блядь вы все лезете, школьники? И самое главное нахуя?! Ну изучайте пока что учебник своего 9 класса. В свое время, в институте вам все расскажут. И про преобразования Лоренца, и про СТО.
Аноним 26/02/15 Чтв 09:45:28 #423 №241950 
>>241929
>Не понятно, почему ты такой вывод сделал?
Посмотрел интерстеллар он, очевидно же. И тоже захотел себя поделить на ноль.
Аноним 26/02/15 Чтв 09:56:01 #424 №241951 
>>241938
Не слушай этого пидора >>241940 . Многомировая интерпретация КМ прекрасно обосновывает существование мультиверса. Единственное, это только множество вселенных все же конечное.
Аноним 26/02/15 Чтв 12:38:12 #425 №241962 
>>241947
Верно.
Аноним 26/02/15 Чтв 12:42:17 #426 №241963 
>>241951
> Многомировая интерпретация КМ прекрасно обосновывает существование мультиверса.
Нет, не обосновывает. Это интерпретация, а не физическая теория, она лишь "меняет шило на мыло" для интуитивного восприятия некоторых квантовых парадоксов (парадоксов с точки зрения "механической" интерпретации квантовых явлений).
Аноним 26/02/15 Чтв 20:39:43 #427 №242058 
>>241963
Но ведь лучше неё ничего не придумано! Копенгагенская интерпретация только лишь постулирует принципиальную неопределенность. Не объясняет откуда эта неопределенность! Вся теория логики построена на детерминизме. Копенгагенская интерпретация недетерминистична, и поэтому не логична.
Аноним 26/02/15 Чтв 21:25:26 #428 №242082 
>>242058
Она неинтуитивна, а не нелогична. Но в науке уже давно перестали стесняться неинтуитивных теорий. "Просто ебашь по формуле, не думай!", как иронично заметил какой-то учёный.
Аноним 26/02/15 Чтв 21:34:35 #429 №242088 
Сорян за лишний тред, не заметил вопросов тред. Не оставляйте без внимания пожалуйста https://2ch.hk/sci/res/242048.html
Аноним 26/02/15 Чтв 23:20:01 #430 №242104 
>>242082
"Заткнись и вычисляй", если быть точнее.
На самом деле довольно идиотский принцип, если класть его в основу научного мировоззрения. Потому что, чтобы что-то вычислить, надо сначала создать теорию - а я что-то не слышал про ученых, которые создавали фундаментальные теории, исходя из чисто позтивистской мотивации "а заебеню-ка такую фундаментальную теорийку чисто ради тупого счета, а какая за этим онтология мира стоит- мне похуй и неинтересно". Даже если такие и были, то их явное меньшинство.
Аноним 26/02/15 Чтв 23:23:43 #431 №242105 
>>242058
>Но ведь лучше неё ничего не придумано!
Ага, не считая того, что механизм выбора ветки истории, в которой оказывается сознание наблюдателя после измерения, по-прежнему подчиняется необъяснимому проекционному постулату. Перекладывание трудности из кармана в карман, обертка ее в другой фантик.
Аноним 27/02/15 Птн 06:49:54 #432 №242124 
Если я вишу на границе чёрной дыры и тыкаю палкой за горизонт событий. Её отрубит или утянет внутрь?
Аноним 27/02/15 Птн 07:03:45 #433 №242128 
>>242124
Тебя размажет в плазму задолго до того, как ты задашь себе этот вопрос.
Аноним 27/02/15 Птн 07:26:15 #434 №242131 
>>242128

Ну ведь есть такая граница где не размажет? Тогда пусть не палка будет в а верёвка длинная.
Аноним 27/02/15 Птн 07:30:06 #435 №242135 
>>242124
с ней произойдет СИНГУЛЯРНОСТЬ, она превратится в СУПЕРПОЗИЦИИ СТРУН, которые КВАНТОВО ТЕЛЕПОРТИРУЮТСЯ в НЕОПРЕДЕЛЕННЫЙ участок прямой кишки твоей мамаши..
Аноним 27/02/15 Птн 07:32:16 #436 №242136 
>>242135

Я думал это тред тупых вопросов, а не обсуждения моей мамаши.
Аноним 27/02/15 Птн 07:39:36 #437 №242139 
>>242124
Тебя и палку разорвет приливными силами и утянет за горизонт событий.
Аноним 27/02/15 Птн 08:06:08 #438 №242142 
>>242105
Нету никакого смысла в выборе ветки истории для конкретного наблюдателя. История реализует все возможные ветки! Рассуждать о конкретном сознании наблюдателя в конкретном мире БЕССМЫСЛЕННО, это просто крайняя степень эгоизма. Да и вообще, что такое "сознание наблюдателя"?
Ты не сможешь ответить на последний вопрос, потому что это такое же бессмысленное понятие, как и механизм выбора ветки истории.
Аноним 27/02/15 Птн 11:36:41 #439 №242161 
>>242139

Я просто представлял чёрную дыру как некую воронку, на "берегу" которой можно было бы порыбачить.
Аноним 27/02/15 Птн 12:26:29 #440 №242174 
>>242131
Граница примерно в пару миллионов световых лет. Такую верёвку не соорудишь, по множеству причин, она превратится в планету или небольшую галактику (не могу прикинуть навскидку).
Аноним 27/02/15 Птн 12:48:54 #441 №242181 
>>242174
>Граница примерно в пару миллионов световых лет
Лолшто.
Аноним 27/02/15 Птн 12:54:11 #442 №242182 
>>241922
1. Нет, чд это скорее колодец, дно которого представляет собой сингулярность, то есть точку в пространстве где все (вообще все), сжато до максимально допустимого размера, ну или до бесконечночсти

2. Если дыра достаточно огромная, можешь хоть на буране туда влететь

3. Ближайшая известная в центре галактики, может есть и ближе, но их еще вычислить надо, они же блядь невидимые! Их еще найти надо
Аноним 27/02/15 Птн 12:58:08 #443 №242184 
>>242142
>Нету никакого смысла в выборе ветки истории для конкретного наблюдателя

Т.е. ты постулируешь, что вопрос о том, исходя из нваблюдатель пронаблюдал показания прибора, отвечающие спину "вверх", а не "вниз", бессмысленен (хотя он почему-то таки пронаблюдал, и другие ветки ему не открылись). Ну ок)
Аноним 27/02/15 Птн 12:59:04 #444 №242186 
>>242182
Хотя не, сравнение с колодцем все таки не корректно, ведь черная дыра это сфера, но суть я пояснил
Аноним 27/02/15 Птн 12:59:06 #445 №242187 
>>242184
...исходя из чего наблюдатель пронаблюдал...
(фикс)
Аноним 27/02/15 Птн 13:10:12 #446 №242191 
>>242128
>>242174
>>242139
Эти неразумные школьники не в курсе, что например у сверхмассивных ЧД приливные силы около горизонта событий значительно слабее. Поэтому человек, находясь в паре метров от горизонта событий, или даже если пересечет его, то ничего не почувствует, не заметит никакой разницы. Объем горизонта событий растет быстрее, чем масса. Масса растет линейно, а объем прямо пропорционален кубу радиуса. Поэтому, чем массивней ЧД, тем более слабые приливные силы около горизонта событий.
Аноним 27/02/15 Птн 13:24:17 #447 №242195 
>>242184
Он бессмысленен относительно функции состояния всего Мультиверса. Выделять конкретное состояние (а ты именно пытаешься выделить конкретное состояние, когда предлагаешь ввести "механизм выбора ветки") не имеет смысла, если реализуются абсолютно все возможные состояния.
В данной интерпретации постулируется относительный характер состояния одной системы по отношению к другой. Все остальное является следствием.
Аноним 27/02/15 Птн 13:42:27 #448 №242201 
>>242191
Нее, сейчас перечитал, то что написал. Короче я тоже хуйню сморозил. Слабые приливные силы не являются следствием кубически возрастающего объема. Интенсивность приливных сил зависит исключительно от радиуса ЧД, а он в свою очередь от массы. Но в целом все верно:
>чем массивней ЧД, тем более слабые приливные силы около горизонта событий.
Аноним 27/02/15 Птн 16:37:16 #449 №242225 
>>241934
Соглашусь, вот по этому и написал сюда, ибо везде инфа очень разрозненная, и многие детали на разных сайтах отличаются. Все еще надеюсь на источники...

>Вот например пещера, в ней на стене висит сосулька

Кстати этим летом был в Абхазии в пещерах и видел все своими глазами, как я понял ты про сталактиты, сталагмиты и сталагнаты? Очень красивое зрелище анон, даже пожить там хотелось.
http://абхазия.рф/отдых-и-туризм/достопримечательности/новый-афон/новоафонская-пещера
Аноним 27/02/15 Птн 19:35:34 #450 №242255 
14250549340790.jpg
>>241603
Большой взрыв НЕ взрыв. Его просто так назвали. Большое раздутие или Большой хлопок не так сексуально звучит вот и прижилось
Аноним 27/02/15 Птн 19:37:05 #451 №242256 
>>241641
>Вселенная описывается современной физикой как расширяющаяся, т.к. по-другому в математике пока нет возможности выразить динамические параметры пространства и материи. Потому такие формулы приводят к такому артефакту как Большой Взрыв, но это просто "предел видимости" описания физических законов, не более того.
у меня от тебя КРАСНОЕ СМЕЩЕНИЕ и реликтовое излучение
Аноним 27/02/15 Птн 19:40:01 #452 №242257 
>>241898
такое название дали неустановленной силе которая серьезно влияет на вращение галактик и что-то там еще

Аноним 27/02/15 Птн 20:04:52 #453 №242264 
Посаны, объясните пожалуйста мне по-проще как-нибудь, что такое сигнатура пространства? Вот только без всяких там квадратичных форм и скалярных произведений это я итак в педивикии прочитал, и нихуя не понял.
Аноним 27/02/15 Птн 20:05:57 #454 №242265 
>>242264
Ну вообще можно объяснить и с квадратичными формами, только и их тогда объясните по понятней.
Аноним 27/02/15 Птн 20:49:49 #455 №242276 
>>236711
Как рассчитать перегрузку?
Вернее интересует скорость, с которой нужно начать вращать 85 кг что бы получить перегрузку "н" G, подразумеваю центрифугу.
Подскажите формулу. А ещё желательно формулу для расчёта оптимального радиуса от оси вращения.
Аноним 27/02/15 Птн 21:00:29 #456 №242278 
>>242276
Школота совсем охуела.
Аноним 27/02/15 Птн 21:02:15 #457 №242280 
>>242278
Причём тут школота, решил собрать на даче центрифугу из старого движка жигуля и палок.
Аноним 27/02/15 Птн 21:16:33 #458 №242281 
>>242280
При том, что ты шкальник, которому нужно решить задачу.
Аноним 27/02/15 Птн 21:20:11 #459 №242283 
>>242281
Ты какой то дурачёк, если бы я был школьник я бы её решил, тк у меня был бы конспект, а так я 27летний долбоёб.
Аноним 27/02/15 Птн 21:47:08 #460 №242290 
>>242256
Это БУДАПЕШТ.
Аноним 28/02/15 Суб 00:18:08 #461 №242318 
>>242264
Ну смари, попробую совсем для чайников.

В твоем уютненьком евклидовом пространстве есть радиус-векторы типа r={x,y,z}. Как бы ты ни вертелся группой поворотов, длина вектора будет постоянна и равна r2=x2+y2+z2. Это типа инвариант. Коэффициентики при иксах-игреках - все положительны (единички). Значит, сигнатура твоего уютненького пространства есть (+ + +). Она "евклидова", и по этой сигнатуре все координаты здесь кагбе равноправны.

Теперь, блять, по хардкору выходишь из этой зоны комфорта и сразу нахуй попадаешь в 4-мерное пространство-время СТО, называемое пространством-временем Минковского. Там уже радиус-вектор, сцуко, будет 4-хкомпонентный (добавляется время): s=(ct,x,y,z). Как бы ты ни вертелся преобразованиями Лоренца, Эйнштейн гарантируэ, что инвариантным теперь будет не обычная "3-мерная" длина или время по отдельности, а т.н. интервал: s2=(ct)2- x2-y2-z2. В СТО именно это - основной инвариант. По сути, это своеобразная "длина" вектора s, но в пространстве Минковского. Так вот, знаки коэффициентиков при координатах здесь такие: (+ - - -). Т.е. имеется один знак и три других (можно было и -+++ написать, зависит от определения). Вот это и есть сигнатура пространства-времени СТО. Она, как говорят, "псевдоевклидова". И она кагбе говорит, что координата, при которой стоит знак отличный от трех остальных (т.е. плюсик), является выделенной и называется времениподобной: именно она играет роль времени.

Ну а вообще-то, конечно, нехуй было всё это городить, а достаточно было просто сказать, что есть такая шняга - метрический тензор пространства-времени, который описывается симметричной матрицей; если последнюю диагонализовать подходящим преобразованием координат, то знаки диагональных компонент и дадут сигнатуру. В частности, в декартовых системах координат это все можно свести к тем коэффициентикам в длинах векторов, про которые выше.
Аноним 28/02/15 Суб 01:20:19 #462 №242328 
>>242195
>если реализуются абсолютно все возможные состояния

В результате чего они реализуются?
Вот есть полный вектор состояния всего универума (и ничего, что хуй знает насколько непротиворечиво его можно ввести), он спокойно себе эволюционирует, допустим, по Шредингеру. Потом, когда пьяный инженегр Вася взглянул на прибор, нам говорят, что внезапно происходит какая-то ветвящаяся НЁХ под названием "все состояния реализуются". Протрезвев и помянув Неймана под картофанчик, Вася сможет совершенно чотко подсчитать вероятность того или иного исхода измерения. В силу каких причин происходит сама вот эта бифуркация, порушившая безмятежную шредингеровскую эволюцию универсума, в результате чего конкретно взятый Вася оказался именно в том-то конкретном манямирке, а не в другом? Да еще с вот этой конкретно вычислимой (и потому нихуя не бессмысленной) вероятностью?..

Все, что многомировая интерпретация может нам на это сказать: мол, забей и не задавайся вопросом, почему ты пронаблюдал именно этот исход, а не другой: на самом деле, с точки зрения универсума, что бы ты ни пронаблюдал, кто-то где-то в другом мире пронаблюдал иначе, и все это одновременно почему-то есть. И ничего что жопой эти миры ты по определению никогда не увидишь и даже не узнаешь, домысел это или нет: чайник Рассела мы ведь тоже не видим - и ничего, летает ведь. И ничего, что ты как наблюдатель движешься по откуда-то взявшейся совершенно конкретной, эмпирически фиксируемой с конкретными вычислимыми вероятностями ветке миров: просто для тебя ну вот так вот так получилось, вот и все. Забей на это, это ведь всё бессмысленно.
Аноним 28/02/15 Суб 05:12:43 #463 №242346 
>>241636
вообщето он есть даже у тебя
когдато видел сюжет про слепого, кторый "клацал" языком и по звуку умел определять примерные размеры и характеристики предметов
Аноним 28/02/15 Суб 10:25:48 #464 №242367 
>>242318
А зачем такое псевдо-пространство придумали? Лучше бы оставались в няшном евклидовом пространчтве и не трогали ни его ни время, посчитав их за инвариант. Легче же считать за инвариант нечто наглядное, а не всякие интервалы, которые даже не представить и не покрутить в воображении. Ведь ОТО это всего лишь, математическое колдунство, вводимое из-за недостаточного знания учОными закономерностей мира. Оно может только приближенно посчитать их фантазии о устройстве мира, но не соответствует реальности.
Аноним 28/02/15 Суб 10:42:03 #465 №242369 
>>241641
Всячески поддерживаю этого просветленного. Добавлю еще, что радиус сферы Хаббла это просто горизонт/граница видимости, дальше которой находятся такие же галактики и простираются они еще очень далеко если не бесконечно или зациклено. Эта граница существует от конечного времени жизни частиц света, старение которых воспринимается как красное смещение, а очень старые угасшие волны с границы воспринимаются как фоновое реликтовое излучение.
Аноним 28/02/15 Суб 12:01:34 #466 №242374 
Излечима ли психопатия? Или нет и таки "ген совести" существует?
Аноним 28/02/15 Суб 14:56:21 #467 №242420 
>>242374
Из неизлечимости психопатии никак не следует то, что существует какой-то связанный с ней ген.
У нас просто нет метода "лечения" этой "болезни", вот и всё.
Аноним 28/02/15 Суб 15:38:49 #468 №242425 
>>242374
>ген совести
а) Почти наверняка гены влияют на личность
б) Генов этих много, а не один
Аноним 28/02/15 Суб 18:13:30 #469 №242452 
>>242425
Не в буквальном смысле, просто тип у психопатов отсутствует или неразвита какая-то хуическая зона в мозге
Аноним 28/02/15 Суб 18:29:35 #470 №242464 
>>242452
Генов как уже сказали много и они влияют опосредовано и еще есть куча всякой сложной нейрохимии.
Ну и вообще для общего развития вот- https://stelazin.livejournal.com/103229.html
Аноним 28/02/15 Суб 22:38:21 #471 №242521 
Давно не был тут, приятно озоснавать, что этот тред живёт и здравствует.

У меня есть два несвязанных между собой вопроса:
1. Как долго длится реакция Белоусова — Жаботинского и почему она заканчивается? Что получается в итоге?
2. Наткнулся на статью, в которой описывается оптический аналоговый компьютер, способный решать задачу коммивояжора в графе буквально за полиномиальное время.
http://ieeexplore.ieee.org/xpl/login.jsp?tp=&arnumber=6801271
Учитывая, что любая NP-полная задача может быть приведена к данной, то это звучит заманчиво. Правда, учитывая максимально возможную энергию луча, которую способно передать волокно и минимальную чувствительность приёмника, возможности данного компьютера ограничиваются несколькими сотнями вершин. Много это или мало? Какой рекорд графа, для которого найден кратчайший путь с помощью традиционных компьютеров?
Аноним 28/02/15 Суб 22:48:23 #472 №242523 
Я щас случайно в /mg заглянул первый раз и таки сказать что я охуел это ни чего не сказать. Какие то тульпы, астралы, чакры, духи ну ебана мать, неужели это не считается психическим отклонением? Я имею ввиду веру в это. В связи с этим вопрос, что заставляет людей верить в подобное? А почитав их я смею догадываться, что они на полном серьезе верят в это все. Почему?
Аноним 28/02/15 Суб 22:50:37 #473 №242524 
Я щас случайно в /sci заглянул первый раз и таки сказать что я охуел это ни чего не сказать. Какие то термояды, платья, радиация, генетическая память ну ебана мать, неужели это не считается психическим отклонением? Я имею ввиду веру в это. В связи с этим вопрос, что заставляет людей верить в подобное? А почитав их я смею догадываться, что они на полном серьезе верят в это все. Почему?
Аноним 28/02/15 Суб 23:42:43 #474 №242526 
>>242523
Ты просто себя так воспитал или окружающее тебя общество с узким кругозором тебя воспитало, что мыслишь их стереотипами и не замечаешь ничего вне их. Таких тут называют образованцами.

Тульпа - подробно воображаемое существо, воображается до тех пор, пока воображение не будет происходить автоматом.
Астрал - внешние реальности, по отношению к нашей реальности, реальности сна и воображения.
Духи - астральные сущности.
Чакра - порт соединения реальностей.

>неужели это не считается психическим отклонением?
Смотря от чего. Если от здравого мышления, то не считается. Отклонений больше у тебя.

>что заставляет людей верить в подобное?
Интерес и интуиция.

>Почему?
Потому что тоже недалекие от тебя. Надо не верить, а исследовать и понимать.
Аноним 28/02/15 Суб 23:52:43 #475 №242530 
>>242523
То, что ты на парашу заходил, — это твоя личная печаль. Но вот нахуй ты этого >>242526 с собой привел?
Аноним 01/03/15 Вск 00:01:45 #476 №242535 
Я щас случайно в /b заглянул первый раз и таки сказать что я охуел это ни чего не сказать. Какие то рулеточки, набеги, ятянпруфовнебудет, засмеялся-обосрался, неужели это не считается психическим отклонением? Я имею ввиду все эти поступки. В связи с этим вопрос, что заставляет людей так поступать? А почитав их я смею догадываться, что они на полном серьезе всё это делают. Почему?
Аноним 01/03/15 Вск 00:36:20 #477 №242543 
>>242530
Бля, по ходу за мной как то увязался и его бред я ни сколько не разделяю. Для чего мне весь этот мусор в голове про тульпы и прочее, я тоже понятия не имею. Он нес что то про изучение и познание, но зачем мне забивать голову и тратить время на изучение, какой то ахинеи которая считалась ересью уже в 17 веке. И вобще изучая все подряд, ты вряд ли преуспеешь хоть в чем то, ибо нахватаешь по верхам, тем более такого не нужного говна, которое под собой и научного обоснования то не имеет.
Аноним 01/03/15 Вск 00:41:07 #478 №242544 
>>242535
Ну епта, там то уж совсем дети лютуют, им заняться не чем, да и не интересно им больше ни хера.
Аноним 01/03/15 Вск 01:27:16 #479 №242548 
Я щас случайно в /es заглянул первый раз и таки сказать что я охуел это ни чего не сказать. Какие то лены, риточки, вскрываемся, алисы, неужели это не считается психическим отклонением? Я имею ввиду все эти поступки. В связи с этим вопрос, что заставляет людей так поступать? А почитав их я смею догадываться, что они на полном серьезе всё это делают. Почему?
Аноним 01/03/15 Вск 01:59:30 #480 №242551 
Подкиньте что-нибудь (фильм, статья, книга) простого и свежего по элементарным частицам, нихуя не могу по статьям из википедии разобраться с кварками, лептонами, бозонами и прочим. в школе такому не учили
Аноним 01/03/15 Вск 02:11:27 #481 №242553 
>>242548
Ну в /mg я хоть понял про что там, а /es то что за хуйня? Мультики что ли?
Аноним 01/03/15 Вск 11:33:41 #482 №242595 
2015-02-27 11:45 #432605
xxx: В детстве, когда смотрел первые две части "Чужих", то больше всего переживал за кота Джонса.

А ведь кошки в космосе до сих пор не были? Только собаки, мыши, обезьяны, земноводные\рептилии, птицы...
Аноним 01/03/15 Вск 13:10:54 #483 №242608 
>>242595
У кошек сильно вестибулярный аппарат развит, отрыв от гравитации для них как отрыв от реальности.
Аноним 01/03/15 Вск 14:05:49 #484 №242614 
Поясните, пожалуйста, как работает т.н. «квантовое бессмертие»? Как я понял, эта штука спасёт наблюдателя, пытающегося застрелиться, выбирая для него продолжение существования в одном из вариантов развития событий, где он остаётся в живых (много миров и все такое). Но что, если наблюдатель заранее обречен? Например, он родился на такой-то планете 20 лет назад, успел засычевать и обзавестись бородой. Но на расстоянии 21 световой год ебанула сверхновая с гамма-всплеском, ровно нацеленным на планету сыча. И вот в 21 год он должен умереть внезапной смертью со всей планетой от страшного потока излучения. До этого события они об этом ничего не знают - событие гамма-всплеска до них ещё не дошло. Как же квантовому бессмертию спасти сыча?
Аноним 01/03/15 Вск 14:29:02 #485 №242617 
>>242614
Очень просто- сыч будет жить в том варианте, где сверхновая не ебнет.
>До этого события они об этом ничего не знают
Кто они, лол? Причем тут вообще чье- то знание или не знание о событии? Ты просто живешь в том мире, где ты жив.
Меня, например, всегда смущало в квантовом бессмертиии другое- ладно там внезапная смерть и все такое, но вот старость- как с ней быть? Как оно может спасти от смерти "от старости"?
Аноним 01/03/15 Вск 14:42:47 #486 №242626 
>>242617
>Как оно может спасти от смерти "от старости"?
а) Квантовое бессмертие никого не "спасает".
б) Смерть от старости — это общее название. У смерти всегда есть конкретные причины, которые и могут не возникнуть/не добить тебя.
Аноним 01/03/15 Вск 14:45:50 #487 №242628 
>>242617
>Очень просто- сыч будет жить в том варианте, где сверхновая не ебнет.
То есть квантовое бессмертие как бы ЗАРАНЕЕ охватывает всё многообразие миров-вариантов? А как же недетерминированность, кукареку, и так далее? Хуёвенько же выходит, раз так.
>Кто они, лол? Причем тут вообще чье- то знание или не знание о событии?
Они - ерохи, соседи сыча по планете, да и сам сыч-наблюдатель, в чьем личном манямирке всё это как-то рефлексирует. Я тут немного сдолбоёбил со "знанием". Лучше было бы сказать "неизбежное событие, которое ещё не наступило и не имеет предвестников к нему". Мгновенная смерть как бы из ниоткуда, распространяющаяся вместе с конусом события, ну.
>старость- как с ней быть? Как оно может спасти от смерти "от старости"?
Квантовое бессмертие тогда по-твоему варианту должно как-то предоставить сычу-наблюдателю средство от старости, опять же ЗАРАНЕЕ. Ведь он
>будет жить в том варианте, где
изобрели лекарство от старости. Мутная тема короч.
Аноним 01/03/15 Вск 15:21:18 #488 №242637 
>>242626
>У смерти всегда есть конкретные причины, которые и могут не возникнуть/не добить тебя
Это- то ясно, но одно дело- тебя застрелят/не застрелят на улице, а другое- старение. Старение не предоставляет тебе варианта умереть/не умереть.
>квантовое бессмертие как бы ЗАРАНЕЕ охватывает всё многообразие миров-вариантов?
Оно ничего не охватывает, фишка в том, что ты просто живешь в том мире, где ты жив. Т.е. что- то вроде разновидности "Когда мы есть, то смерти еще нет, а когда смерть наступает, то нас уже нет".
Аноним 01/03/15 Вск 15:25:48 #489 №242639 
>>242637
Ну по твоему тогда ты будешь жить в мире, где старости нет же.
Аноним 01/03/15 Вск 15:38:00 #490 №242650 
>>242639
Звучит как какая- то хуйня, если честно. Как я понимаю все- таки многообразие миров-вариантов задается многообразием возможных квантовых состояний, а не многообразием того, что вообще можно придумать. Т.е. законы природы- физика, биология итп в мирах будут одинаковыми.
Аноним 01/03/15 Вск 18:13:42 #491 №242704 
>>236711
Тупой вопрос Существует ли топология пространств, т.е. измерений.
Аноним 01/03/15 Вск 21:58:34 #492 №242792 
>>242704
Ну вообще да, в математике существуют топологические пространства. Пространство Минковского, к примеру, является топологическим векторным пространством.
Аноним 02/03/15 Пнд 13:26:38 #493 №242944 
14252919987710.png
>>241254
  ▲
▲ ▲
Аноним 02/03/15 Пнд 15:01:54 #494 №242976 
>>242944

▲ ▲
(Автор этого поста был забанен. Помянем.)
Аноним 02/03/15 Пнд 15:44:51 #495 №242991 
>>242944

▲ ▲
(Автор этого поста был предупрежден.)
Аноним 02/03/15 Пнд 20:23:13 #496 №243072 

▲ ▲
Аноним 04/03/15 Срд 01:41:09 #497 №243536 
14254224696680.jpg
*ПОСОНЫ!!! У меня вопрос! Какого хуя люди говорят про полупрозрачное стекло/линзу/зеркало? Неужели оно и правда ПОЛУпрозрачное? Я, например, не верю, что можно сделать такое зеркало, которое будет пропускать ровно 50% света, а не 49,99999999999999999999999999999%. Поясните. Неужели всем похуй на то, что оно на самом деле не точно полупрозрачное. Или оно и правда полупрозрачное? Тогда как люди добиваются такой высочайшей точности? Как создаются такие зеркала?
Аноним 04/03/15 Срд 04:36:45 #498 №243576 
>>243536
А что, полудурок это ровно 49.999999999 процента от дурака?
Аноним 04/03/15 Срд 05:01:47 #499 №243579 
>>243576
>А что, полудурок это ровно 49.999999999 процента от дурака?
Нет, это ровно 50% же.
Аноним 04/03/15 Срд 08:37:13 #500 №243595 
14254474333950.jpg
Анон, я тут задался внезапно таким вопросом.
Вот смотри - мы видим какую-нибудь там звезду, которая находится в миллионе световых лет, т.е. мы видим ее состояние по отношению к тому времени, что было для нас 1кк лет назад.
Так вот, допустим человечество придумало телепорт/гипердвигатели, чтобы мгновенно переместиться туда, к этой звезде, и у людей есть координаты звезды в полярной системе координат, и этого достаточно (т.е. мы можем указать двигателю "прыгай туда, на такое-то расстояние").
По получается так, что прыгнув к нужному месту звезды там не окажется! Ибо она за 1кк лет уже успела переместиться в другое место. И оказавшись в новом месте, мы увидим свет искомой нашей звезды, но он опять будет "старым", излученным звездой когда-то там давно.
И сколько так придется скакать до того, пока корабль не достигнет самой звезды? Или же все решается простой корректировкой?
Аноним 04/03/15 Срд 09:45:57 #501 №243606 
>>243595
Такие мгновенные скачки невозможны. Они противоречат всем общепризнанным научным теориям и приводят к причинно-следственному парадоксу. Твой мысленный эксперимент не имеет смысла.
Аноним 04/03/15 Срд 09:51:53 #502 №243608 
>>243606
А кротовые норы? Или иные варианты путешествия через 4е измерение к нужной точке?
Общепризнанные теории не равны аксиомам из разряда "через две точки можно провести только одну прямую".
Аноним 04/03/15 Срд 09:54:00 #503 №243610 
>>243595
Ну это же геометрический вопрос, а не физический. Да, можно будет корректировать путь, предсказывая, во что превратилась какая-то область неба, если сейчас она вот так отображает прошлое. Наверное, миллионы лет уже делают предсказания невозможными (слишком точность моделей/вычислений низкая для таких цифр), а может, миллиарды.

Да, озарение пришло, пока думал об этом. Получается, звёздное небо большей частью вообще показывает шум, который соответствует реальности лишь приблизительно, стохастически, так как отражает объекты, расстояние до которых больше, чем срок их жизни. К примеру, вон там мы видим скопление звёзд, формирующее галактику, и мы сможем обнаружить эту галактику, телепортировавшись туда, потому что галактика имеет большой лайф-цикл, больше, чем у звёзд. А вот тех звёздных систем, которые нарисованы на нашем небе в этой галактике, вообще не обнаружим. Эдакая квантовая механика огромных расстояний.

Надо будет уточнить в википедии сроки жизни планет, звёзд, галактик, скоплений галактик, аттракторов и их геометрические размеры, чтобы определить границы вселенной, за которой она становится принципиально (физически) случайной.
Аноним 04/03/15 Срд 10:06:24 #504 №243617 
>>243608
Кротовые норы это всего лишь одно из предельных следствий ОТО. Т.е. этим обозначается граница применимости ОТО. Тоже самое как и черная дыра. Суть этого в том, что ОТО предсказывает возможность существования таких объектов, но принципиально не может их описать. Это говорит о том, что ОТО не полна. Некоторые ученые убеждены, что через кротовые норы могут проходить только элементарные частицы, кванты, и это не будет приводить к нарушению причинно-следственных связей.
Аноним 04/03/15 Срд 12:37:17 #505 №243653 
>>243536
ПОСОНЫ, НУ БАМП ЖЕ! ЭТО СЕРЬЕЗНЫЙ ВОПРОС КАК БЕ! ВЫ ВАЩЕ В КУРСЕ НА КАКОМ ПРИНЦИПЕ ПОСТРОЕНЫ КВАНТОВЫЕ ГЕНЕРАТОРЫ СЛУЧАЙНОСТИ? ИМЕННО НА ЁБАНОМ ПОЛУПРОЗРАЧНОМ ЗЕРКАЛЕ. ИЛИ ВЫ ХУЕСОСЫ НИХУЯ НЕ СМЫСЛЯЩИЕ В НАУКЕ ТУТ СОБРАЛИСЬ? МАМАШУ ВАШУ ЕБАЛ В ТАКОМ СЛУЧАЕ.
Аноним 04/03/15 Срд 13:48:32 #506 №243672 
>>243595
И как ты собрался определить эти границы?
Определишь - отпиши хотя-бы.
Аноним 04/03/15 Срд 13:59:14 #507 №243674 
>>243672
Какие границы, ты точно мне хотел ответить? (>>243595-кун)

>>243617
Бро, ты бы еще к орфографии приебался, ну честное слово. Ясное дело что вероятность существования кротовых нор, прохождение сквозь них людей и кораблей малы и т.п. Но ебана, мы же рассматриваем вопрос на гране реальности и допущений ("а может быть..."). Может по сути есть чего?
Аноним 04/03/15 Срд 14:01:38 #508 №243675 
>>243617
опять же, нарушение какой причинно-следственной связи ты подразумеваешь?
Аноним 04/03/15 Срд 14:04:09 #509 №243676 
>>243674
>>243672 Переадресовано >>243610-куну
Аноним 21/03/15 Суб 18:34:58 #510 №249524 
>>236714
>а может ли в дальнейшем появится еще какое- нибудь взаимодействие?
Может. Электромагнитное же разделили на магнитное и электрическое.
comments powered by Disqus

Отзывы и предложения